EAQ Assignemnt #2

¡Supera tus tareas y exámenes ahora con Quizwiz!

The nurse is caring for a patient who is receiving an inspired oxygen concentration of more than 60%. What value of arterial O2 (PaO2) would indicate hypoxemic respiratory failure? Record your answer using a whole number.

Hypoxemic respiratory failure is commonly defined as a PaO2 less than 60 mm Hg when the patient is receiving an inspired oxygen concentration of 60% or more. This definition incorporates two important concepts. First, the PaO2 level indicates inadequate oxygen saturation of hemoglobin; second, this PaO2 level exists despite administration of supplemental oxygen at a percentage (60%) that is about three times that in room air (21%).

The nurse is assessing a patient who smokes and notes gray-colored sputum with specks of brown. What action would be appropriate for this patient? 1. No action is required, as it is considered normal. 2. Administer oxygen therapy to the patient. 3. Assess oxygen saturation through pulse oximetry. 4. Inform the health care provider

1 RATIONALE Gray-colored sputum with specks of brown is a normal finding in an individual who smokes, so no action is required. Administering oxygen therapy, assessing oxygen saturation, and informing the health care provider are not necessary for this patient.

The nurse cares for an immunocompetent patient. Which clinical manifestation is most indicative of pulmonary tuberculosis? 1. Mucopurulent sputum 2. Diarrhea and fatigue 3. Lymph node enlargement 4. Hematuria and dehydration

1 RATIONALE: A cough that progresses in frequency and produces mucoid or mucopurulent sputum is the most common symptom of pulmonary tuberculosis (TB). Diarrhea, and hematuria, and dehydration are manifestations not directly associated with pulmonary TB. Fatigue and lymph node enlargement may be seen with TB but are not as indicative as is the production of mucopurulent sputum.

Please place options in order of importance. 1.Explain procedure to patient 2.Place tracheostomy care kit or appropriate sterile equipment at bedside 3.Place patient in semi-Fowler position 4.Put on goggles and clean gloves 5.Auscultate chest sounds 6.Change tracheostomy tubes

The procedure must first be explained to the patient. The proper equipment can be placed at the bedside. If the equipment is placed first, the patient might be frightened. Position the patient in semi-Fowler position. Next put on goggles and clean gloves and auscultate chest sounds before beginning the procedure to determine if suctioning is needed. Next change the tracheostomy cannula. After the tracheostomy has been changed and dressing applied, tracheostomy ties can be changed.

The nurse is monitoring a patient following lung transplantation for manifestations of acute rejection, such as which of the following? Select all that apply. 1. Fatigue 2. High fever 3. Low-grade fever 4. Productive cough 5. Oxygen desaturation

1,3,5 RATIONALE: Acute rejection is fairly common in lung transplantation, typically occurring in the first 5 to 10 days after surgery. Low-grade fever, fatigue, dyspnea, dry cough, and oxygen desaturation are signs of rejection. High fever and productive cough are not signs of acute rejection.

A patient began taking antitubercular drugs a week ago. The nurse reviews the patient's medical record and learns that the patient has a 10-year history of consuming one standard drink of alcohol three times a week. The patient states, "In the last week, my urine turned orange and I am very worried about it." How should the nurse respond? 1 Inform the patient that it is one of the side effects of the antitubercular drug rifampin (Rifadin). 2 Recognize that the tuberculosis may have spread to the liver, and further medical consultation is required. 3 Recognize that the liver may be damaged due to alcohol, and so a liver function test should be performed. 4 Instruct the patient to stop taking antitubercular drugs immediately and consult the primary health care provider.

1 RATIONALE: A nurse should be aware of some of the common side effects of antitubercular drugs like rifampin, one of which is orange discoloration of body fluids such as urine, sweat, tears, and sputum. It may also cause hepatitis. Liver damage can lead to jaundice, which usually presents as yellowish discoloration of urine and sclera. However it is highly unlikely that tuberculosis has spread to the liver. The alcohol intake of the patient is within normal limits, and so it is not correct to say that alcohol may have damaged the liver. It is also inappropriate to advise the patient to stop taking antitubercular drugs.

To maintain patient safety, the nurse would question the health care provider about the prescription for prednisone if the patient also had which condition? 1 Systemic fungal infection 2 Diabetes mellitus 3 Congestive heart failure 4 Renal insufficiency

1 RATIONALE: Systemic fungal infection would be a contraindication to the use of prednisone because the drug can interfere with the body's ability to fight infection. While blood sugars may increase and fluid retention may occur, diabetes, congestive heart failure, and renal insufficiency are not absolute contraindications to the use of prednisone, although it should be used with caution.

When teaching the patient about reducing the risks of lung cancer, what is the most important topic for the nurse to address? 1. Smoking cessation 2. Drinking more fluids 3. Eating more grains 4. Using steam inhalation

1 RATIONALE: Lung cancer and the risks associated with it can be reduced by smoking cessation, and the patient should be counseled to discontinue use of tobacco and avoid secondhand smoke as much as possible. Drinking more fluids, eating more grains, and using steam inhalation are general measures to promote health and do not have much impact on reducing risks of lung cancer.

When admitting a patient with a diagnosis of asthma exacerbation, the nurse will assess for what potential triggers? Select all that apply. 1, Exercise 2, Allergies 3. Emotional Stress 4. Decreased humidity 5. Upper respiratory infections

1, 2, 3, 5 RATIONALE: Although the exact mechanism of asthma is unknown, there are several triggers that may precipitate an attack. These include allergens, exercise, air pollutants, upper respiratory infections, drug and food additives, psychologic factors, and gastroesophageal reflux disease (GERD). Decreased humidity is not a trigger.

The nurse is caring for the patient with a pulmonary embolism. Which factor(s) are associated with a pulmonary embolism (PE)? Select all that apply. 1. Pregnancy 2. Pelvic surgery 3. Immobility 4. Herbal therapy

1,2,3 RATIONALE: Pregnancy, pelvic surgery, and immobility are major risk factors associated with a pulmonary embolism. Risk factors among many for PE include immobility, pelvic surgery, pregnancy, oral contraceptives and hormone therapy. Herbal therapy is incorrect because herbal therapy is not associated with the development of a pulmonary embolism.

Pulmonary rehabilitation (PR) is an evidence-based intervention that includes many disciplines working together to individualize treatment of the symptomatic chronic obstructive pulmonary disease (COPD) patient. What is PR designed to do? Select all that apply. 1. Reduce symptoms 2. Improve quality of life 3. Reduce effort by teaching to inhale when pushing 4. Enhance rest by teaching to exhale with relaxation 5. Provide a "last ditch" effort for patients with COPD

1,2 RATIONALE: PR is an evidence-based intervention that includes many disciplines working together to individualize treatment of the symptomatic COPD patient. PR is designed to reduce symptoms and improve quality of life. PR is an effective intervention to improve exercise capacity and decrease hospitalizations, anxiety, and depression. An energy-saving tip is to exhale when pushing, pulling, or exerting effort during an activity and inhale during rest. PR should no longer be viewed as a "last ditch" effort for patients with severe COPD.

A patient with a nasal fracture is scheduled for a fracture reduction surgery in two weeks. What preoperative instructions should a nurse give to the patient? Select all that apply. 1. Avoid nonsteroidal antiinflammatory drugs (NSAIDs). 2. Refrain from smoking. 3. Limit exercising. 4. Avoid ice application on the fractured area. 5. Do not consume alcoholic drinks for one week before the surgery

1,2 RATIONALE: The nurse should instruct the patient to avoid intake of NSAIDs at least two weeks before the surgery to reduce the risk of bleeding. The patient should avoid smoking, as smoking cessation promotes better wound healing. Avoiding exercise is a postoperative instruction and helps to prevent bleeding. Applying ice is a postoperative instruction to decrease swelling. Alcohol tends to increase swelling and should be avoided after the operation.

While recording respirations of a patient, what assessment findings would a nurse report as paradoxic breathing? Select all that apply. 1. Chest moving inward during inspiration 2. Abdomen moving inward during inspiration 3. Dyspnea on lying down 4. Dyspnea upon standing 5. Use of pursed-lip breathing

1,2 RATIONALE: Normally, the thorax and abdomen move outward on inspiration and inward on exhalation. During paradoxic breathing, the abdomen and chest move inward during inspiration due to maximal use of the accessory muscles of respiration. Dyspnea on standing and lying down are not paradoxic breathing. Pursed-lip breathing helps to prevent collapse of the airways and is also not paradoxic breathing.

The nurse is caring for a patient with stage III laryngeal cancer that is scheduled to undergo a total laryngectomy. To assist the patient to reestablish the ability to speak, which voice rehabilitation measures should be taken pre- and postoperatively? Select all that apply. 1. Preoperatively, a speech therapist should be consulted. 2. Transesophageal puncture can be considered at the time of laryngectomy. 3. Speaking tracheostomy tubes can be placed. 4. Radiation therapy can be planned. 5. Sign language can be used.

1,2 RATIONALE: A speech therapist should be asked to meet the patient to educate and discuss options for speech and voice restoration after surgery. Transesophageal puncture is the most common and practical voice restoration option and can be performed at the time of laryngectomy. Speaking tracheostomy tubes and radiation therapy are not useful for patients who do not have vocal cords. Sign language education would help in communication but not in restoring voice.

On auscultation of patient's lungs, which breath sounds would the nurse consider normal? Select all that apply. 1. Loud, high-pitched sounds resembling air blowing through a hollow pipe 2. Soft, low-pitched, gentle, rustling sounds heard over all lung areas except the major bronchi 3. Medium-pitched sounds heard anteriorly over the mainstem bronchi on either side of the sternum 4. The patient repeats the phrase "ninety-nine," and the words are easily understood and are clear and loud through the chest. 5 The patient whispers "one-two-three," and the almost inaudible voice is transmitted clearly and distinctly.

1,2,3 RATIONALE Bronchial, vesicular, and bronchovesicular sounds are normal breath sounds. Bronchial sounds are loud and high-pitched and resemble air blowing through a hollow pipe. Vesicular sounds are soft, low-pitched, gentle, rustling sounds heard over all lung areas except the major bronchi. Bronchovesicular sounds are medium-pitched sounds heard anteriorly over the mainstem bronchi on either side of the sternum. Bronchophony is an abnormal breath sound and is considered positive (abnormal) if the patient repeats the phrase "ninety-nine," and the words are easily understood and are clear and loud. Whispered pectoriloquy is also an abnormal breath sound and is considered positive (abnormal) when the patient whispers "one-two-three," and the almost inaudible voice is transmitted clearly and distinctly.

The nurse checks for which abnormal physical assessment findings consistent with cor pulmonale? Select all that apply. 1. Jugular vein distention 2. Pedal edema 3. Crackles 4. Wheezing 5. Hepatomegaly

1,2,5 RATIONALE: Cor pulmonale is a right-sided heart failure caused by resistance to right ventricular outflow caused by lung disease. With failure of the right ventricle, the blood emptying into the right atrium and ventricle would be slowed, leading to jugular vein distension, hepatomegaly, and pedal edema. Crackles would be associated with left-sided heart failure. Wheezing is not present in either type of heart failure.

To promote airway clearance in a patient with pneumonia, what should the nurse instruct the patient to do? Select all that apply. 1. Maintain adequate fluid intake 2. Splint the chest when coughing 3. Maintain a 30-degree elevation 4. Maintain a semi-Fowler's position 5. Instruct patient to cough at end of exhalation

1,2,5 RATIONALE: Maintaining adequate fluid intake liquefies secretions, allowing easier expectoration. The nurse should instruct the patient to splint the chest while coughing. This will reduce discomfort and allow for a more effective cough. Coughing at the end of exhalation promotes a more effective cough. The patient should be positioned in an upright sitting position (high-Fowler's) with head slightly flexed.

The nurse understands that in normal lungs the volume of blood perfusing the lungs each minute (4 to 5 L) is approximately equal to the amount of gas that reaches the alveoli each minute (4 to 5 L). In a perfectly matched system, each portion of the lung would receive 1 mL of air (ventilation) for each 1 mL of blood flow (perfusion). What would be the V/Q in this case? Record your answer using a whole number.

1:1 --> V/Q = 1 A match of ventilation and perfusion would result in a V/Q ratio of 1:1, which is expressed as V/Q = 1. When the match is not 1:1, a V/Q mismatch occurs.

A patient with an acute exacerbation of chronic obstructive pulmonary disease (COPD) needs to receive precise amounts of oxygen. Which equipment should the nurse prepare to use? 1 Oxygen tent 2 Venturi mask 3 Nasal cannula 4 Oxygen-conserving cannula

2 The Venturi mask delivers precise concentrations of oxygen and should be selected whenever this is a priority concern. An oxygen tent is not required for a patient with COPD. A nasal cannula delivers a less-precise amount of oxygen. An oxygen-conserving cannula is not appropriate for this patient. Text Reference - p. 590

The nurse provides home care instructions to a patient who has emphysema. Which statement by the patient indicates correct understanding of the teaching? 1. "I'll get a dehumidifier to use at home." 2. "I'll report any change in the color of my sputum." 3. "When I feel short of breath, I'll increase my oxygen flow rate." 4. "The correct procedure for pursed-lip breathing is to inhale twice as long as I exhale."

2 RATIONALE: A change in the color of the patient's sputum from clear to yellow or green may indicate an infection and therefore should be reported to the patient's healthcare provider. Patients with emphysema should have humidified air in the home to prevent drying of the respiratory tract. The rate of oxygen delivery should not be increased over that recommended by the primary healthcare provider because of the risk of decreased respiratory drive. The correct procedure for pursed-lip breathing is to exhale twice as long as the breather inhales.

A patient diagnosed with acute respiratory distress syndrome is being mechanically ventilated with 12 cm of positive-end expiratory pressure (PEEP). Upon assessment, the nurse notes deterioration of vital signs and absent breath sounds in the right lung field. What is the most likely cause for this finding? 1. Deterioration of the disease 2. Pneumothorax 3. Decreased cardiac output 4. Obstructed endotracheal tube

2 RATIONALE: A complication of PEEP may be a pneumothorax as a result of overdistention of the alveoli. If deterioration of the disease were the cause, both lung sounds would be decreased equally. Decreased cardiac output would affect vital signs, but not breath sounds. An obstructed endotracheal tube would affect both lung fields.

CHART/EXHIBITION 1 A patient is concerned that he or she may have asthma. The nurse assesses the severity of the symptoms. Based on this information, how would the nurse classify the patient's asthma? Refer to chart. 1 Intermittent severity 2 Mild persistent 3 Moderate persistent 4 Severe persistent

2 RATIONALE: This patient's asthma would be classified as "mild persistent" severity. The patient reports having symptoms more than twice a week, but not daily, wakes up with symptoms at night three or four times a month, uses a short-acting beta agonist more than twice a week, but not daily, and reports a minor limitation in normal activity. The lung function assessment shows FEV1 over 80% and normal FEV1/FVC.

A patient whose tracheostomy was inserted 30 minutes ago is recovering in the postanesthesia recovery unit when the patient coughs and expels the tracheostomy tube. How should the nurse respond? 1. Suction the tracheostomy opening 2. Maintain the airway with a sterile hemostat 3. Use an Ambu bag and mask to ventilate the patient 4. Insert the tracheostomy tube obturator into the stoma

2 RATIONALE: As long as the patient is not in acute respiratory distress after dislodging the tracheostomy tube, the nurse should use a sterile hemostat to maintain an open airway until a sterile tracheostomy tube can be reinserted into the tracheal opening. If the patient is in respiratory distress, the nurse will use an Ambu bag and mask to ventilate the patient temporarily. The tracheostomy is an open surgical wound that has not had time to mature into a stoma.

A patient diagnosed with acute respiratory distress syndrome is being mechanically ventilated with 12 cm of positive-end expiratory pressure (PEEP). Upon assessment, the nurse notes deterioration of vital signs and absent breath sounds in the right lung field. What is the most likely cause for this finding? 1 Deterioration of the disease 2 Pneumothorax 3 Decreased cardiac output 4 Obstructed endotracheal tube

2 A complication of PEEP may be a pneumothorax as a result of overdistention of the alveoli. If deterioration of the disease were the cause, both lung sounds would be decreased equally. Decreased cardiac output would affect vital signs, but not breath sounds. An obstructed endotracheal tube would affect both lung fields. Text Reference - p. 1663

A patient had an intradermal tuberculin skin test (Mantoux) administered 48 hours ago. The nurse assesses the injection site and identifies a 12-mm area of palpable induration. How should the nurse interpret this result? 1.Definitive evidence that the patient does not have tuberculosis 2. A significant indication that the patient has been exposed to tuberculosis 3. Delayed hypersensitivity with a high likelihood of infection with tuberculosis 4. A negative test that cannot be interpreted as ruling out the presence of tuberculosis

2 RATIONALE: An area of 12 mm of induration at the injection site 48 hours after a Mantoux test is considered significant for a past or current tuberculin infection. An induration of less than 5 mm is considered a negative result. The other answer options are incorrect conclusions related to the findings.

patient with cystic fibrosis is being treated by the nurse with chest physiotherapy for the left upper lobe (LUL) of the lung. In which order should the nurse perform the following actions? Put a comma and space between each answer choice (1, 2, 3, 4). 1.Effective coughing with expectoration 2.Albuterol (Proventil) nebulizer therapy 3.Application of percussion and vibration 4.Position patient on right side with head up

2,1,3,4 RATIONALE: Chest physiotherapy for the LUL begins with a short-acting bronchodilator to facilitate ventilation during therapy. After bronchodilator use, the patient is positioned on the right side with the head up to expose the LUL for treatment and drainage. Percussion and vibration are performed to loosen secretions. Then the nurse should assist the patient with coughing effectively and expectorating the thick secretions.

The nurse is caring for a patient with respiratory failure. What information should the nurse include when explaining the disease condition to the family members? Select all that apply. 1 Respiratory failure is inevitable after cardiac failure. 2 Respiratory failure is categorized as hypoxemic or hypercapnic. 3 Respiratory failure is insufficient oxygen transfer into the blood. 4 Respiratory failure is inadequate carbon dioxide removal from the lungs. 5 Respiratory failure is a disease that presents with various respiratory symptoms.

2,3,4 RATIONALE: Respiratory failure is classified as hypoxemic or hypercapnic. Respiratory failure results when gas exchanging functions are inadequate, i.e., insufficient oxygen is transferred to the blood or inadequate carbon dioxide is removed from the lungs. Although respiratory failure may be secondary to cardiac failure, not all cases of respiratory failure are secondary to cardiac failure. Respiratory failure is not a disease but a symptom of an underlying pathologic condition affecting lung function, oxygen delivery, cardiac output, or the baseline metabolic state.

A patient with acute respiratory distress syndrome (ARDS) has been put on a ventilator. What nursing measures alter the risk of development of pneumonia secondary to ventilator use? Select all that apply. 1 Using invasive monitoring devices 2 Maintaining proper hygiene of the patient 3 Keeping the patient on prolonged ventilation 4 Using aseptic techniques when suctioning the patient 5 Using the enteral route for feeding the patient

2,4 RATIONALE: Maintaining proper hygiene of the patient and using aseptic techniques may decrease the risks of pneumonia in the patient caused by the ventilator. The use of invasive monitoring devices and keeping the patient on prolonged ventilation increase the risk of pneumonia by facilitating passage of microorganisms into the airway. When a patient is on ventilator and on enteral feedings, there is a high risk of aspiration of feeds, causing pneumonia.

A nurse observes a patient with asthma using a metered-dose inhaler. The nurse should correct which patient actions? Select all that apply. 1 The patient inspires slowly. 2 The patient holds the breath for 5 seconds. 3 The patient shakes the device before using it. 4 The patient presses the meter dose inhaler gently. 5 The patient coordinates the use of the inhaler with inspiration.

2,4 RATIONALE: While using a metered dose inhaler, the breath should be held for at least 10 seconds for better absorption of the drug. The metered dose inhaler should be pressed with adequate strength to deliver the required dose. Inspiring slowly, shaking the device before using it, and coordinating use of the inhaler with inspiration are the right techniques for using a metered dose inhaler

A patient is being discharged from the emergency department after being treated for epistaxis. In teaching the family first aid measures in the event the epistaxis would recur, what measures should the nurse suggest? Select all that apply. 1 Tilt patient's head backwards 2 Apply ice compresses to the nose 3 Tilt head forward while lying down 4 Pinch the entire soft lower portion of the nose 5 Partially insert a small gauze pad into the bleeding nostril

2,4,5 RATIONALE: First aid measures to control epistaxis include tilting the patient's head backwards and placing the patient in a sitting position, leaning forward. Pinching the soft lower portion of the nose or inserting a small gauze pad into the bleeding nostril should stop the bleeding within 15 minutes. Tilting the head back or forward does not stop the bleeding, but rather allows the blood to enter the nasopharynx, which could result in aspiration or nausea/vomiting from swallowing blood. Lying down also will not decrease the bleeding.

A pediatric patient is diagnosed with pertussis. The cough is worse during the night and is accompanied by a loud, long, rasping, indrawn breath. Even though the child has completed a week's course of antibiotics and has received cough suppressants, the cough has worsened. The nurse determines that what is the likely reason the cough has worsened? 1. The child may have lung cancer. 2. It could be a side effect of the antibiotics. 3. It may be due to administration of cough suppressant. 4. It could be a fungal infection.

3

A nurse preparing educational information about lung cancer notes that the primary risk factor related to the development of lung cancer is 1 Genetics 2 Chewing tobacco 3 Cigarette smoking 4 Occupational exposure

3 As many as 90% of patients with lung cancer have a history of cigarette smoking. Cigarette smoke contains several organ-specific carcinogens. Genetics and occupational exposure are risks but not as high as cigarette smoking. Chewing tobacco is not associated with lung cancer; it is related to oral cancer. Text Reference - p. 535

Which clinical manifestation should the nurse expect to find during assessment of a patient admitted with pneumonia? 1. Hyperresonance on percussion 2. Vesicular breath sounds in all lobes 3. Increased vocal fremitus on palpation 4. Fine crackles in all lobes on auscultation

3 RATIONALE: A typical physical examination finding for a patient with pneumonia is increased vocal fremitus on palpation. Other signs of pulmonary consolidation include bronchial breath sounds, egophony, and crackles in the affected area. With pleural effusion, there may be dullness to percussion over the affected area.

The nurse is caring for a patient admitted with a barbiturate overdose. The patient is comatose with blood pressure (BP) 90/60, apical pulse 110, and respiratory rate 8. Based upon the initial assessment findings, the nurse recognizes that the patient is at risk for which type of respiratory failure? 1 Hypoxemic respiratory failure related to shunting of blood 2 Hypoxemic respiratory failure related to diffusion limitation 3 Hypercapnic respiratory failure related to alveolar hypoventilation 4 Hypercapnic respiratory failure related to increased airway resistance

3 The patient's respiratory rate is decreased as a result of barbiturate overdose, which caused respiratory depression. The patient is at risk for hypercapnic respiratory failure resulting from the decreased respiratory rate, and thus decreased CO2 elimination. Barbiturate overdose does not lead to shunting of blood, diffusion limitations, or increased airway resistance.

A nurse is caring for a patient diagnosed with acute lung injury who has a PaO2 of 48 mm Hg. Which condition does the nurse expect? 1.Dysoxia 2.Hypoxia 3.Hypoxemia 4.Hyperventilation

3 RATIONALE: Hypoxemia, an inadequate amount of oxygen in the blood, frequently is quantified as a PaO2 of less than 50 mm Hg. If allowed to progress, hypoxemia can result in hypoxia, which is defined as an inadequate amount of oxygen available at the cellular level such that cells experience anaerobic metabolism. Dysoxia is a condition characterized by an inability of the cells to use oxygen properly despite adequate levels of oxygen delivery. There is not enough information to determine if the patient is hyperventilating.

The various events occurring in the exudative phase of acute respiratory distress syndrome (ARDS) are listed. Arrange these events in the sequential order of development. 1. Necrotic cells, protein, and fibrin form hyaline membranes that line the alveoli. 2. Fluid from the interstitial space crosses the alveolar membrane and enters the alveolar space. 3. Engorgement of the peribronchial and perivascular interstitial space produces interstitial edema. 4. Damage to alveolar type I and II cells is followed by atelectasis due to reduced surfactant synthesis.

3, 2, 4, 1 RATIONALE: The exudative phase of acute respiratory distress syndrome (ARDS) occurs within 1 to 7 days after the initial direct lung injury. Initially, engorgement of the peribronchial and perivascular interstitial space produces interstitial edema. Next, fluid from the interstitial space crosses the alveolar membrane and enters the alveolar space. Decreased synthesis of surfactant and inactivation of existing surfactant cause the alveoli to become unstable and collapse (atelectasis). Finally, necrotic cells, protein, and fibrin form hyaline membranes that line the alveoli.

Bed rest is prescribed for a patient during the acute phase of respiratory failure. What is the rationale for the recommendation of bed rest and limitation of other activity in the plan of care? 1 To prevent further alveolar collapse 2 To decrease the basal metabolic rate 3 To promote the clearance of secretions 4 To reduce the cellular demand for oxygen

4 Respiratory failure interferes with ventilation and oxygenation. It is essential to reduce the body's need for oxygen at the cellular level. Bed rest is an essential and effective means of reducing the need for oxygen. Bed rest and limitation of activity do not prevent alveolar collapse, clear secretions, or decrease the basal metabolic rate. Text Reference - p. 1659

A patient had an open reduction repair of a bilateral nasal fracture. The nurse plans to implement an intervention that focuses on both nursing and medical goals for this patient. Which intervention should the nurse implement? 1. Apply an external splint to the nose 2. Insert plastic nasal implant surgically 3. Humidify the air for mouth breathing 4. Maintain surgical packing in the nose

4 RATIONALE: A goal that is common to nursing and medical management of a patient after rhinoplasty is to prevent the formation of a septal hematoma and potential infections resulting from a septal hematoma. Therefore, the nurse helps to keep the nasal packing in the nose. The packing applies direct pressure to oozing blood vessels to stop postoperative bleeding. A medical goal includes realigning the fracture with an external or internal splint. The nurse helps maintain the airway by humidifying inspired air because the nose is unable to do so following surgery because it is swollen and packed with gauze.

A patient is in acute respiratory distress syndrome (ARDS) as a result of sepsis. Which measure most likely would be implemented to maintain cardiac output? 1 Administer crystalloid fluids or colloid solutions. 2 Position the patient in the Trendelenburg position. 3 Place the patient on fluid restriction and administer diuretics. 4 Perform chest physiotherapy and assist with staged coughing.

1 Low cardiac output may necessitate crystalloid fluids or colloid solutions in addition to lowering positive end-expiratory pressure (PEEP) or administering inotropes. The Trendelenburg position (not recommended to treat hypotension) and chest physiotherapy are unlikely to relieve decreased cardiac output, and fluid restriction and diuresis would be inappropriate interventions. Text Reference - p. 1669

The nurse cares for a patient with a diagnosis of tuberculosis. Which assessment finding best indicates that the patient has been following the prescribed treatment plan? 1 Negative sputum cultures 2 Clear breath sounds bilaterally 3 Decrease in the number of coughing episodes 4 Conversion of the Mantoux test from positive to negative

1 RATIONALE: A patient's sputum is expected to convert to negative within 3 months of the beginning of treatment. If it does not, the patient is either not taking the medication or has drug-resistant organisms. Bilaterally clear breath sounds and a decrease in coughing are good indications that the patient is following the prescribed plan, but they are not as confirmatory as negative sputum cultures. Once a person has been exposed to the tuberculosis-causing organism, the Mantoux test will always elicit a positive result.

The nurse is caring for an 83-year-old patient and is reviewing laboratory results. The PaO2 is 76 mm Hg. What nursing action is appropriate for this finding? 1 Nothing, as this is a normal finding for this patient. 2 Notify respiratory therapy to prepare for intubation. 3 Notify the primary health care provider immediately. 4 Administer oxygen via nasal cannula at 2 L/minute

1 RATIONALE: Age affects normal arterial blood gas (ABG) values. Elderly people have a lower PaO2 level, with a decrease in approximately 10 mm Hg per decade. Normal ABG values are ranges for normal, healthy adults. It is important to establish a baseline for the individual because abnormal values become "normal" for some individuals. Notifying either respiratory therapy or the primary health care provider is not necessary at this time. Administering oxygen is not necessary at this time.

Which inhaler should the nurse be prepared to administer to the patient at the onset of an asthma attack? 1 Albuterol (Proventil) 2 Fluticasone/Salmeterol (Advair) 3 Fluticasone (Flovent) 4 Salmeterol (Serevent)

1 RATIONALE: Albuterol is a short-acting bronchodilator that should be given first when the patient experiences an asthma attack. Fluticasone/salmeterol, fluticasone, and salmeterol are not short-acting bronchodilators and will not relieve the patient's symptoms of an acute asthma exacerbation.

The nurse measures the pulse of a patient admitted with bronchitis and asthma and finds it to be 120 beats per minute. To which medication might the nurse attribute this finding? 1 Albuterol (Proventil HFA) 2 Montelukast (Singulair) 3 Acetaminophen (Tylenol) 4 Allopurinol (Zyloprim)

1 RATIONALE: Albuterol, used to treat asthma, increases the sympathetic nervous system response, and may increase the heart rate. Allopurinol, acetaminophen, and montelukast do not elevate the heart rate.

The nurse is caring for a patient admitted to the hospital with pneumonia. Upon assessment, the nurse notes a temperature of 101.4° F, a productive cough with yellow sputum, and a respiratory rate of 20. Which most appropriate nursing diagnosis is based upon this assessment? 1. Hyperthermia related to infectious illness 2. Ineffective thermoregulation related to chilling 3. Ineffective breathing pattern related to pneumonia 4. Ineffective airway clearance related to thick secretions

1 RATIONALE: Because the patient has spiked a temperature and has a diagnosis of pneumonia, the logical nursing diagnosis is hyperthermia related to infectious illness. There is no evidence of a chill, and the patient's breathing pattern is within normal limits at 20 breaths/minute. There is no evidence of ineffective airway clearance from the information given because the patient is expectorating sputum.

A patient with chronic obstructive pulmonary disorder is considered for lung transplantation. The patient had melanoma and hepatitis A one year ago. The patient previously smoked three cigarettes per day but has not smoked in the past year. The nurse considers the patient's history and concludes what about the patient's eligibility for lung transplantation? 1. The patient is eligible for lung transplantation. 2. Because of the history of smoking, the patient is not eligible. 3. Because of the history of hepatitis A, the patient is not eligible. 4. Because of the history of cancer one year ago, the patient is not eligible.

1 RATIONALE: Chronic obstructive pulmonary disease is one of the indications for lung transplantation. There are some absolute contraindications for lung transplantation, including but not limited to being a current smoker, chronic active hepatitis B or C, history of cancer except skin cancer, poor nutritional status, and HIV. This patient is eligible for lung transplantation. The patient is not a current smoker, has hepatitis A, and had melanoma, which is a skin cancer. Therefore there are no contraindications for lung cancer.

The nurse notices clear nasal drainage in a patient newly admitted with facial trauma, including a nasal fracture. What should the nurse do first? 1. Test the drainage for the presence of glucose 2. Suction the nose to maintain airway clearance 3. Document the findings and continue monitoring 4. Apply a drip pad and reassure the patient that this is norma

1 RATIONALE: Clear nasal drainage suggests leakage of cerebrospinal fluid (CSF). The drainage should be tested for the presence of glucose, which would indicate the presence of CSF. Suctioning should not be done. Documenting the findings and monitoring are important after notifying the health care provider. A drip pad may be applied, but the patient should not be reassured that this is normal.

A patient with chronic obstructive pulmonary disease (COPD) is suspected to have developed cor pulmonale. The nurse recognizes that which test result helps confirm the diagnosis? 1 Large pulmonary vessels on chest X-ray 2 Decreased pressure found in a right heart catheterization 3 Left-sided heart enlargement on echocardiogram 4 Decreased B-type natriuretic peptide (BNP) levels

1 RATIONALE: Cor pulmonale is a cardiac complication of COPD resulting from pulmonary hypertension. Due to pulmonary hypertension, the pulmonary vessel may appear enlarged in a chest X-ray. There may be increased pressure found in a right heart catheterization due to pulmonary hypertension. Cor pulmonale is usually associated with right-sided heart enlargement, as there is increased pressure in the blood vessels of lungs. The BNP levels are increased due to the stretching of the right ventricle.

A nurse has taught the technique of pursed-lip breathing to a patient. During a return demonstration, what patient action requires correction? 1. Puffing of cheeks while exhaling air 2. Slow and deep inhalation through the nose 3. Slow exhalation through pursed lips, as if whistling 4. Exhalation time three times as long as inhalation time

1 RATIONALE: In pursed-lip breathing, the patient should avoid puffing of the cheeks while exhaling the air. Puffing of the cheeks makes the technique less effective. Slow and deep inhalation, slow exhalation through pursed lips as if whistling and exhalation time thrice as long as inhalation time are correct techniques of pursed-lip breathing.

The patient has been receiving oxygen by nasal cannula. The nurse suspects the patient is experiencing oxygen toxicity after noting which finding? 1. Restlessness 2. Tachypnea 3. Diminished lung sounds 4. Oxygen saturation 98%

1 RATIONALE: Oxygen toxicity is rare; signs and symptoms include restlessness, confusion, and substernal chest pain. Tachypnea, decreased lung sounds, and low oxygen saturation are not consistent with oxygen toxicity.

The nurse is assessing a patient with a respiratory problem. Which findings indicate the patient is suffering from pulmonary fibrosis? 1. Normal percussion 2.Prolonged expiration 3. Fremitus over affected area 4. Egophony over effusion

1 RATIONALE: Patients with pulmonary fibrosis have normal percussion findings. Inspection would reveal tachypnea and palpation would show movement. Auscultation shows crackles or sounds like Velcro being pulled apart. Prolonged expiration occurs with asthma and fremitus over affected area occurs with pneumonia. Egophony over effusion often occurs with pleural effusion.

The nurse is caring for a patient who is suspected of having chronic obstructive pulmonary disease (COPD). The nurse knows that what type of diagnostic test would confirm this diagnosis? 1. Spirometry 2. Chest x-ray 3. Arterial blood gas (ABG) 4. Computed tomography (CT) scan of the ches

1 RATIONALE: Spirometry is needed to confirm the presence of airflow obstruction and the severity of COPD. The patient is given a short-acting bronchodilator, and postbronchodilator values are compared with a normal reference value. Chest x-rays are not diagnostic but can show a flat diaphragm caused by hyperinflated lungs. ABGs are used after the patient has been diagnosed and usually in the more severe stages of the disease. CT scans are not used routinely to diagnose COPD.

The nurse is caring for the patient with a productive cough. The nurse collects a sputum specimen for an acid-fast bacillus (AFB) smear. What collection time by the nurse is most appropriate? 1. 6 AM 2. 12 noon 3. 6 PM 4. 9 PM

1 RATIONALE: The correct answer is 6 AM because if the patient has a productive cough, early morning is the ideal time to collect sputum specimens for an AFB smear because secretions collect during the night. Twelve noon, 6 PM, and 9PM are incorrect because all of these times are afternoon or evening hours and the amount of secretions for the specimen may not be optimal.

The nurse would determine that levofloxacin (Levaquin) therapy has not been effective after noting which indicator? 1. White blood cell count 14,000/mm3 2. Temperature 99.7° F 3. Increased respiratory rate 4. Adventitious lung sounds

1 RATIONALE: The normal white blood cell (WBC) count is 5000 to 10,000/mm3. An elevated WBC count above the normal range indicates persistence of the infection during treatment with an antibiotic. Lung sounds and respiratory rate are not indicators of the efficacy of antibiotic treatment. A low grade fever also may signify that the infection is persisting; however, the white blood cell count is the most reliable indicator of active infection.

The nurse is teaching a patient how to use a hand-held nebulizer. Which guideline is correct? 1 Sit in an upright position during the treatment. 2 Take short, shallow breaths while inhaling the medication. 3 Rinse the nebulizer equipment under running water once a week. 4 During the treatment, breathe in and hold the breath for five seconds.

1 RATIONALE: The patient is placed in an upright position that allows for most efficient breathing to ensure adequate penetration and deposition of the aerosolized medication. The patient must breathe slowly and deeply through the mouth and hold inspirations for two or three seconds. Deep diaphragmatic breathing helps ensure deposition of the medication. Instruct the patient to breathe normally in between these large forced breaths to prevent alveolar hypoventilation and dizziness. After the treatment instruct the patient to cough effectively. An effective home-cleaning method is to wash the nebulizer equipment daily in soap and water, rinse it with water, and soak it for 20 to 30 minutes in a 1:1 white vinegar-water solution, followed by a water rinse and air drying.

A male patient with chronic obstructive pulmonary disease (COPD) becomes dyspneic at rest. His baseline blood gas results are PaO2 70 mm Hg, PaCO2 52mm Hg, and pH 7.34. What updated patient assessment requires the nurse's priority intervention? 1. Arterial pH 7.26 2. PaCO2 50 mm Hg 3. Patient in tripod position 4. Increased sputum expectoration

1 RATIONALE: The patient's pH shows acidosis that supports an exacerbation of COPD along with the worsening dyspnea. The PaCO2 has improved from baseline, the tripod position helps the patient's breathing, and the increase in sputum expectoration will improve the patient's ventilation.

When caring for a patient with pertussis, the nurse will prioritize 1. Administering antibiotic therapy 2. Having the patient use an antihistamine at night 3. Teaching the patient how to use a bronchodilator 4. Instructing the patient to use cough suppressants

1 RATIONALE: Treatment for pertussis is antibiotics, usually macrolides (erythromycin, azithromycin [Zithromax]), to minimize symptoms and prevent spread of the disease. Cough suppressants and antihistamines should not be used, because they are ineffective and may induce coughing episodes. Corticosteroids and bronchodilators are not useful in reducing symptoms.

When initially teaching a patient the supraglottic swallow following a radical neck dissection, with which food or fluid should the nurse begin? 1. Cola 2. Applesauce 3. French fries 4. White grape juice

1 RATIONALE: When learning the supraglottic swallow, it may be helpful to start with carbonated beverages because the effervescence provides clues about the liquid's position. Nonpourable pureed foods, such as applesauce, would decrease the risk of aspiration, but carbonated beverages are the better choice with which to start. French fries would not be easy to swallow when learning the supraglottic swallow. Thin, watery fluids should be avoided because they are difficult to swallow and increase the risk of aspiration.

A nurse is caring for a patient with bronchitis. The health care provider has advised arterial blood gas (ABG) analysis. The blood sample can be obtained from which blood vessels? Select all that apply. 1 Radial artery 2 Femoral artery 3 Pulmonary artery 4 Superior vena cava 5 Femoral vein

1,2 RATIONALE: Blood samples for ABG analysis are collected from the radial and femoral arteries, as they carry oxygenated blood. The pulmonary artery carries deoxygenated blood and is used for the measurement of mixed venous blood gas. The superior vena cava is used to measure the central venous pressure (CVP). The femoral vein is not used for ABG analysis, since it carries deoxygenated blood

When teaching patients about preventing occupational lung diseases, what instructions should the nurse include? Select all that apply. 1 Use effective ventilation systems. 2 Ensure periodic inspection of workplaces. 3 Wear masks for high-risk occupations. 4 Avoid exposure to affected coworkers. 5 Use steam inhalation directly after working in a dusty environment.

1,2,3 RATIONALE Occupational lung diseases result from inhaled dust or chemicals and can be prevented by using effective ventilation systems, periodically inspecting workplaces to ensure safety, and wearing masks for high-risk occupations. It is not necessary to avoid coworkers who have been infected with lung diseases, and steam inhalation is not an effective deterrent.

A patient presents with acute exacerbation of asthma. The nurse expects that which strategies will be included in the treatment plan? Select all that apply. 1 Administration of 100% oxygen 2 Nebulization with short-acting β2-adrenergic agonists (SABAs) 3 Intravenous administration of corticosteroids 4 Administration of sedatives 5 Administration of antibiotics

1,2,3 RATIONALE: Acute exacerbation of asthma may be life-threatening and needs immediate intervention. Administering 100% oxygen helps to relieve hypoxia and improve tissue oxygenation. Nebulization with SABA helps to relax the airways and promote airflow. Corticosteroids are administered to blunt the hyperactive immune response. Sedatives should be avoided as they may depress the respiratory center and worsen dyspnea. Antibiotics are not administered unless there are symptoms of pneumonia.

The patient is receiving 3 L of oxygen (O2) via nasal cannula. Which action by the nurse is most appropriate? Select all that apply. 1 Assesses the bubble-through humidifier if humidity is used. 2 Assures that the patient is wearing the nasal cannula correctly 3 Adjusts humidification according to patient comfort. 4 Realizes that humidification is never needed.

1,2,3 RATIONALE: Assessing the bubble-through humidifier if humidity is used, assuring that the patient is wearing the nasal cannula correctly, and adjusting humidification according to patient comfort are correct because oxygen (O2) obtained from cylinders or wall systems is dry. Dry O2 has an irritating effect on mucous membranes and dries secretions. A common device used for humidification when the patient has a cannula or a mask is a bubble-through humidifier. It is important for the nurse to assess the bubble-through humidifier if humidity is used to make sure the humidification is on. This adds to the comfort of the patient. The nurse assesses the patient to make sure the nasal cannula is worn correctly for optimal effect. The cannula can become easily dislodged. Humidification is adjusted according to the patient's comfort level. When oxygen levels are 1-4 L the use of humidification may not be the preference of all patients. Realizing that humidification is never needed is incorrect because the use of humidification is patient preference.

A patient is admitted to the hospital with respiratory failure. Which drugs should the nurse anticipate to be advised for the patient? Select all that apply. 1 Diltiazem (Cardizem) for atrial fibrillation 2 Metoprolol (Lopressor) for atrial fibrillation 3 Azithromycin (Zithromax) to treat infections 4 Lorazepam (Ativan) for pulmonary congestion 5 Fentanyl (Sublimaze) for sedation and diuresis

1,2,3 RATIONALE: Diltiazem (Cardizem) and metoprolol (Lopressor) relieve pulmonary congestion in atrial fibrillation by decreasing heart rate and improving cardiac output. Azithromycin (Zithromax) is an antibiotic used to treat infections. Lorazepam (Ativan) is a benzodiazepine and does not improve pulmonary congestion. Fentanyl (Sublimaze) is a good opioid analgesic used to relieve pain, but it does not cause sedation or diuresis.

The nurse makes a nursing diagnosis of "impaired gas exchange" for a patient with pneumonia based upon which physical-assessment findings? Select all that apply. 1 SpO2 of 85% 2 PaO2 65 mm Hg 3 Absent breath sounds in right lung lobes 4 Presence of thick yellow mucus 5 Respiratory rate 24 breaths/minute

1,2,3 RATIONALE: Impaired gas exchange is evidenced by low oxygen saturation and elevated PaCO2 with absent breath sounds. Yellow mucus would indicate clearance of secretions. An increased respiratory rate does not imply impaired gas exchange.

A patient has benzodiazepines (lorazepam [Ativan]) and opioids (morphine) ordered to decrease anxiety, agitation, and pain. What are the nursing roles in managing this patient? Select all that apply. 1 Monitoring cardiac function 2 Monitoring respiratory function 3 Ensuring regular sedation holiday 4 Using opioids aggressively as they reduce the length of hospital stay 5 Increasing the dose of morphine if respiration is depressed

1,2,3 RATIONALE: The nurse has to monitor patients closely for cardiopulmonary depression when giving benzodiazepines such as lorazepam (Ativan) and opioids (morphine). Patients receiving these agents are best managed by following an evidence-based protocol that includes a regular "sedation holiday" for ongoing assessment. Sedative and analgesic agents may have a prolonged effect in critically ill patients, delay weaning from mechanical ventilation, and contribute to increased length of stay. If respirations become depressed, then the dose of morphine has to be reduced.

A patient presents with epistaxis. Which interventions are appropriate to control the bleeding? Select all that apply. 1. Reassure the patient and keep him quiet. 2. Place the patient in a sitting position with his head tilted forward. 3. Apply direct pressure by pinching the entire soft lower portion of the nose. 4. Administer saline nasal sprays to relieve congestion. 5. Ask the patient to blow his nose to remove all the collected blood.

1,2,3 RATIONALE: To control epistaxis, the patient should be reassured and kept quiet. In epistaxis, approximately 90% of nosebleeds occur in the anterior portion of the nasal cavity and can be easily visualized. The patient should be made to sit, leaning slightly forward, with the head tilted forward. Direct pressure should be applied by pinching the entire soft lower portion of the nose against the nasal septum for 10 to 15 minutes. If bleeding does not stop within 15 to 20 minutes, consult the health care provider. Saline nasal sprays should not be used as these can dislodge the clot that is needed to stop the bleeding. Nose blowing will also remove the clot, which could lead to further bleeding.

When educating a patient about managing sinusitis without pharmacological interventions, which instructions should the nurse include? Select all that apply. 1. Use a steam inhaler. 2. Sleep with your head elevated. 3. Restrict fluid intake. 4. Avoid exposure to smoke. 5. Apply a cold compress on your cheeks.

1,2,4 RATIONALE: In the case of sinusitis, steam inhalation helps to promote drainage of secretions. Sleeping with the head elevated helps to drain the sinuses and reduce congestion. Smoke is an irritant and will worsen the symptoms of sinusitis. Adequate fluid intake will decrease the symptoms of sinusitis. Applying a cold compress on the cheeks is not recommended, as this worsens the symptoms. A hot compress on the cheeks will help.

The nurse is caring for the patient with pulmonary hypertension. Which treatment(s) are appropriate? Select all that apply. 1. Vasodilators 2. Diuretics 3. Anticoagulants 4. Thrombolytics

1,2,3 RATIONALE: Vasodilators, diuretics, and anticoagulants are correct because they are included in drug treatments for pulmonary hypertension. Vasodilators are especially important in the treatment of pulmonary hypertension as it will aid in reducing the right ventricular workload by dilating pulmonary vessels. Diuretics decrease plasma volume and thereby reduce myocardial workload. Anticoagulants also are used, especially if the case is severe, as it works to prevent in situ thrombus formation and venous thrombosis. Warfarin would be given to keep the international normalized ratio (INR) in the 2 to 3 range. Thrombolytic therapy is not an appropriate drug treatment and would be used if the condition causes right ventricle hypertrophy, resulting in cor pulmonale.

When caring for a patient with tuberculosis, what measures should the nurse instruct the patient to take to avoid the spread of infection? Select all that apply. 1. Cover the nose and mouth with a tissue while coughing and sneezing. 2. Throw used tissues in a paper bag and dispose with the trash. 3. Carefully wash hands after handling sputum and soiled tissues. 4. Wear a standard isolation mask when outside the patient's room. 5. Get out of bed and move freely about the hospital to keep up strength. 6. Drink plenty of water and maintain an erect posture.

1,2,3,4 RATIONALE: In order to prevent the spread of infection, patients with tuberculosis should be encouraged to cover the nose and mouth with tissues while coughing and sneezing, to throw used tissues in a paper bag and dispose of them with the trash, to carefully wash hands after handling sputum and soiled tissues, and to wear a standard isolation mask while moving out of their room. Increasing the frequency of prolonged visits to other parts of the hospital is not advisable, as it can increase the chances of infection spread; instead, such visits should be limited. Drinking plenty of water and maintaining erect posture have no effect on controlling infection.

A patient who has bronchiectasis asks the nurse, "What conditions would warrant a call to the clinic?" Which of these would the nurse include in the answer? Select all that apply. 1. Chest pain 2. Fever and chills 3. Blood clots in the sputum 4. Increased sputum production 5. Cough with no sputum production

1,2,3,4 RATIONALE: Teach the patient and caregiver to recognize significant clinical manifestations to report to the health care provider. These manifestations include increased sputum production, bloody sputum, increasing dyspnea, fever, chills, and chest pain. The patient will not have a nonproductive cough.

The nurse is caring for a patient who is receiving continuous oxygen (O2) therapy. The nurse knows that the method of O2 administration for the patient depends upon which factors? Select all that apply. 1. Financial resources 2. Patient's cooperation 3. Comfort of the device 4. Patient's cultural status 5. Humidification required 6. Fraction of inspired oxygen (FiO2) required

1,2,3,5,6 RATIONALE: The method of oxygen administration depends upon the patient's fraction of inspired oxygen (FiO2) needed, the mobility status, the level of humidification needed, resources available, level of cooperation, and comfort of the device. The patient's cultural status does not influence the type of O2 device used.

The nurse is discharging a patient with chronic obstructive lung disease (COPD) who will be attending an outpatient pulmonary rehabilitation (PR) program. The nurse knows that components of this type of program generally include which of the following? Select all that apply. 1. Education 2. Exercise training 3; Smoking reduction 4. Nutrition counseling 5. On-site breathing treatments 6. Attendance by at least one family member or significant other

1,2,4 RATIONALE: Nutrition counseling definitely is needed in a PR program to teach the patient healthy nutrition. Education is also necessary, as well as exercise training. Smoking cessation and not reduction is needed. On-site breathing treatments are not offered. A family member or significant other is not a required component of these programs.

The nurse is teaching a class about smoking cessation. Select the respiratory-related symptoms associated with cigarette smoking. Select all that apply. 1. Chronic cough 2. Decreased sense of taste 3. Decreased sputum production 4. Paralysis of the cilia inside the lungs 5. Increased function of alveolar macrophages

1,2,4 RATIONALE: The effects of cigarette smoking on the respiratory system include development of a chronic cough, paralysis of the cilia, decreased sense of taste and smell, increased sputum production (not decreased), and decreased (not increased) function of alveolar macrophages.

While obtaining a health history for a patient with suspected tuberculosis, the nurse expects the patient to report which signs or symptoms? Select all that apply. 1. Night sweats 2. Fatigue 3. Chest tightness 4. Fever 5. Dyspnea

1,2,4 RATIONALE: The patient with tuberculosis presents with night sweats, fatigue, and fever. Chest tightness and dyspnea are not present in the patient with tuberculosis.

A patient in respiratory distress has thick and viscous secretions that are difficult to expel. Which nursing interventions can help to overcome this? Select all that apply. 1 Adequate fluid intake (2 to 3 L/day) 2 Oxygen given by aerosol mask 3 Acetylcysteine (Mucomyst) without a bronchodilator 4 Hydration through intravenous (IV) fluids 5 Aerosols of sterile normal saline, administered through a nebulizer

1,2,5 RATIONALE: Adequate fluid intake (2 to 3 L/day) keeps secretions thin and easier to remove. Oxygen may also be given by aerosol mask to thin secretions and facilitate their removal. Aerosols of sterile normal saline, administered by a nebulizer, may be used to liquefy secretions. Mucolytic agents such as nebulized acetylcysteine (Mucomyst) should be mixed with a bronchodilator, because they cause bronchoconstriction. If the patient is unable to take sufficient fluids orally, IV hydration is used with thorough assessment of patient's cardiac and renal status.

A patient reports a headache, nasal congestion, and fever for the past three days. A nurse examines the patient's nose and sinus areas thoroughly. What findings would suggest that the patient has sinusitis? Select all that apply. 1 Hyperemic and edematous mucosa and hyperemia 2 Tenderness over the sinuses 3 Clear nasal discharge 4 Nosebleed 5 Swollen turbinates

1,2,5 RATIONALE: Clinical findings that indicate sinusitis include hyperemic and edematous mucosa, tenderness over the involved sinuses, and enlarged turbinates. The inflammation results in increased blood supply to the affected area, which leads to hyperemic and edematous mucosa. Due to the inflammation, there may be tenderness over the involved frontal and/or maxillary sinuses. The turbinates may enlarge due to congestion. Clear nasal discharge is not a sign of sinusitis. Patients with sinusitis usually have a purulent nasal discharge. Nosebleed is not a manifestation of sinusitis as it is associated with epistaxis.

A nurse is caring for a patient with pneumonia. The nurse is most likely to auscultate what breath sounds when assessing the patient's lungs? Select all that apply. 1 Egophony 2 Wheezes 3 Bronchophony 4 Stridor 5 Whispering pectoriloquy

1,3,5 RATIONALE: Pneumonia will present with egophony, bronchophony, and whispering pectoriloquy. Egophony is a test to assess breath sounds. It is positive when the patient is asked to pronounce "E" but instead says "A." In bronchophony, the patient is asked to repeat "ninety-nine" several times in a row. If the words are easily understood and are clear and loud, it indicates an abnormal finding. In pectoriloquy, the patient is asked to whisper "one-two-three." If the whisper is heard clearly and distinctly, it indicates an abnormal finding. Wheezes are heard in asthma when there is bronchoconstriction. Stridor is heard in laryngeal diseases due to the obstruction of the larynx or trachea.

The nurse is caring for a patient with a pleural effusion. What nursing interventions are appropriate when preparing this patient for a thoracentesis? Select all that apply. 1 Ensure that the informed consent was signed. 2 Position patient upright with elbows on an overbed table with feet supported. 3 Instruct the patient to cough vigorously during the procedure. 4 Instruct the patient not to eat anything 4 hours before the procedure. 5 Obtain chest x-ray after the procedure.

1,2,5 RATIONALE: For a thoracentesis, the nurse should ensure that the patient's informed consent was signed. The patient should be positioned upright with elbows on an overbed table and feet supported. This position gives appropriate access for needle insertion. A chest x-ray is obtained after the procedure to rule out a pneumothorax. The patient should be instructed not to talk or cough during the procedure, as it can cause injury by displacement of the needle. NPO status, or withholding food and drink, is not required for thoracentesis.

A patient is diagnosed with a pneumothorax, and the health care provider has inserted a chest tube with chest drainage system. The nurse who is monitoring the system finds that there is no bubbling. The nurse checked all the connections and found no problems. What is the most probable reason for the absence of bubbling? Select all that apply. 1. Suctioning is not present. 2. Suction pressure is very high. 3. Suction pressure is very low. 4. Pleural air leak is too large to be drained. 5. There is collection of blood in pleural space.

1,3,4 RATIONALE: It is important for a nurse to keep the water at the appropriate level in a suction chamber. If there is no bubbling seen, it indicates that there is no suction, or suction pressure is not enough, or the pleural leak is too large to be drained by the given suction pressure. The nurse should therefore revise the suction pressure. Collection of blood in pleural space would be evident by the type of drainage.

A nurse observes a patient using a dry powder inhaler device. The nurse should correct which patient actions? Select all that apply. 1. The patient breathes into the inhaler. 2. The patient performs deep and quick breathing. 3. The patient inhales more than 1 puff with each inspiration. 4. The patient shakes the medicine before using it. 5. The patient holds the breath for more than 10 seconds

1,3,4 RATIONALE: When using a dry powder inhaler, the patient should not breathe into the inhaler as this affects the dosing. Inhaling more than one puff with each inspiration may cause waste of the medication. The patient should not shake the medicine before using it as it can alter the dosing. Deep and quick breathing is the proper technique as it ensures that the medicine moves deep into the lungs. The patient should be encouraged to hold the breath beyond 10 seconds to help in penetration of the dry powder.

When admitting a 45-year-old female with a diagnosis of pulmonary embolism, the nurse will assess the patient for which risk factors? Select all that apply. 1. Obesity 2. Pneumonia 3. Malignancy 4. Cigarette smoking 5. Prolonged air travel

1,3,4,5 RATIONALE: An increased risk of pulmonary embolism is associated with obesity, malignancy, heavy cigarette smoking, and prolonged air travel with reduced mobility. Other risk factors include deep vein thrombosis, immobilization, surgery within the last three months, oral contraceptives and hormone therapy, heart failure, pregnancy, and clotting disorders. Pneumonia is not a risk factor.

A nurse is monitoring a patient with respiratory failure. What are the possible complications of hypoxemia and hypoxia which the nurse should be aware of? Select all that apply. 1 Fall in blood pressure (BP) 2 Hyperdynamic state of liver 3 Hyperdynamic state of heart 4 Metabolic alkalosis and cell death 5 Permanent brain damage

1,3,5 RATIONALE: BP and cardiac output can fall, and vasoactive or inotropic agents are often less effective in an acidotic environment due to hypoxemia and hypoxia. A cardiovascular hyperdynamic state may occur due to catecholamine release, which is associated with the physiologic stress response. Permanent brain damage may occur if the hypoxia is severe and prolonged. Hypoxemia does not lead to a hyperdynamic state of the liver. If hypoxia or hypoxemia is severe, the cells shift from aerobic to anaerobic metabolism. The waste product of anaerobic metabolism is lactic acid. When the body does not have enough sodium bicarbonate to buffer the lactic acid, metabolic acidosis, tissue and cellular dysfunction, and cell death may occur.

A patient is diagnosed with pulmonary embolism. What nursing actions are appropriate for this patient? Select all that apply. 1. Administer oxygen therapy as prescribed. 2. Keep the patient on bed rest in a supine position. 3. Maintain an IV line for medications and fluid therapy. 4. Instruct the patient not to cough or perform deep breathing. 5. Monitor the patient for complications of anticoagulant therapy

1,3,5 RATIONALE: Pulmonary embolism requires prompt therapy for a good prognosis. Oxygen therapy should be administered as prescribed. An IV line should be maintained for medications and fluid therapy. Anticoagulants and fibrinolytics may have adverse effects, and the nurse should monitor the patient for side effects. The patient should be placed in a semi-Fowler's position to assist in breathing. The patient should be encouraged to cough and perform deep breathing exercises for better pulmonary function.

A patient with allergic rhinitis has been advised to use beclomethasone (Beconase) nasal spray to relieve his symptoms. What instructions should be given to this patient about the use of corticosteroid nasal spray? Select all that apply. 1. Use the spray twice daily, as ordered. 2. Use the spray as needed. 3. Discontinue use if nasal infection occurs. 4.Start two weeks before pollen season begins. 5. Clear nasal passages before using the spray.

1,3,5 RATIONALE: The corticosteroid nasal spray should be used on a regular basis to obtain maximum benefit. In this case, the patient should take nasal spray twice daily, as ordered. Use of the nasal spray should be discontinued if nasal infection occurs, as it may suppress the immune system and aggravate the infection. Nasal passages should be cleared before using the spray to ensure that the medicine reaches the target area. The spray should not be used on an as-needed basis, as overuse can worsen symptoms. The spray does not need to be started two weeks before the pollen season begins.

The nurse is assessing a patient who was admitted from a nursing home with suspected tuberculosis (TB). Which of these are initial manifestations of tuberculosis? Select all that apply. 1. Anorexia 2. Dyspnea 3. Night sweats 4. Hemoptysis 5. Low-grade fever 6. Unexplained weight loss

1,3,5,6 RATIONALE: Active TB disease may manifest initially with constitutional symptoms such as fatigue, malaise, anorexia, unexplained weight loss, low-grade fevers, and night sweats. Dyspnea is a late symptom that may signify considerable pulmonary disease or a pleural effusion. Hemoptysis, which occurs in less than 10% of patients with TB, is also a late symptom.

A patient with advanced stage chronic obstructive pulmonary disease (COPD) experiences significant weight loss. The nurse expects that what will be included on the patient's plan of care? Select all that apply. 1. Encourage rest before meals. 2. Encourage high fluid intake with food. 3. Discontinue oxygen therapy while eating. 4. Encourage high-protein supplements between meals. 5. Use steroids and theophylline to relieve bloating

1,4 RATIONALE: Decreased appetite and weight loss occur because of systemic inflammatory processes in COPD. Resting before taking meals helps the patient reduce dyspnea and conserve energy. High-protein supplements can be taken between meals to compensate for the catabolic effects of COPD. Fluid intake should be encouraged between meals rather than with food to prevent distension of the stomach. Supplemental oxygen therapy while eating is beneficial to the patient. Steroids and theophylline cause bloating and do not relieve it.

A nurse is assessing the pulmonary functions of a patient and finds that the vital capacity and total lung capacity are reduced. What respiratory disorders could cause the reduced capacities? Select all that apply. 1. Atelectasis 2. Emphysema 3. Chronic bronchitis 4. Pulmonary fibrosis 5. Asthma

1,4 RATIONALE: Pulmonary function tests are used to differentiate restrictive pulmonary disorders from obstructive disorders. Reduction in vital capacity and total lung capacity indicates that the patient has restrictive pulmonary disorder. Both atelectasis and pulmonary fibrosis are examples of restrictive disorders. Emphysema, chronic bronchitis, and asthma are obstructive pulmonary disorders and are highly unlikely to be seen in this patient.

The nurse is teaching a patient how to use an inhaler. What steps should the nurse teach the patient? Please place selected options in sequential order. 1. Shake the inhaler. 2. Keep breathing in slowly and as deeply as possible. 3. Place the inhaler near or in your mouth as instructed by your health care provider (HCP) 4. Breathe out all the way. 5. As you start breathing in slowly through the mouth, press down on the inhaler one time 6. Hold your breath as you count to 10 slowly.

1,4,3,5,2,6 RATIONALE: Shake the inhaler. Next breathe out all the way and then, while holding the inhaler as prescribed by your HCP, place the inhaler in or near your mouth. Breathe in slowly through the mouth and press down on the inhaler one time. Keep breathing in slowly and as deeply as possible. Then hold your breath while counting slowly to 10.

he nurse is caring for a patient with blunt abdominal injuries caused by a motor vehicle accident. The nurse understands that pain in the patient can cause a ventilation/perfusion (V/Q) mismatch leading to respiratory failure. What pathophysiological processes may be involved in V/Q mismatch caused by pain? Select all that apply. 1 The pain interferes with chest and abdominal wall movement. 2 The pain decreases muscle tension. 3 The pain causes systemic vasodilation. 4 The pain activates the stress response. 5 The pain produces generalized muscle rigidity

1,4,5 RATIONALE: Pain interferes with chest and abdominal wall movement and compromises ventilation. It also activates the stress response. Pain results in generalized muscle rigidity. All these activities increase oxygen consumption and carbon dioxide production, increasing the ventilation demands. All these conditions result in limited airflow to the alveoli but have no effect on blood flow to the gas exchange units. The consequence of the imbalance is V/Q mismatch. Pain does not decrease muscle tension; it increases the muscle tension, causing muscle rigidity and an increase in oxygen demand. Pain also causes vasoconstriction rather than vasodilation.

The nurse is suctioning the patient's tracheostomy. Which occurrence is the first priority consideration by the nurse? 1. Heart rate increases from a baseline of 65 to 70 2. Heart rate decreases from a baseline of 65 to 44 3. SpO2 decreases from 100% to 92% 4. SpO2 decreases from 99% to 90%

2 RATIONALE: A heart rate decrease by 20 or more beats from baseline is an indication to immediately discontinue suctioning through the tracheostomy. A heart rate increase from baseline by 40 or more beats is an indication to immediately discontinue suctioning through the tracheostomy. The heart rate only increases by 5 beats and is not a reason, by itself, to discontinue suctioning. A decrease in SpO2 less than 90% is an indication to discontinue suctioning through the tracheostomy.

The nurse is assigned to care for an 83-year-old patient with an acute asthma exacerbation. Which arterial blood gas (ABG) result would prompt the nurse to notify the provider immediately? 1 pH 7.32, PaO2 70 mm Hg, PaCO2 50 mm Hg 2 pH 7.30, PaO2 74 mm Hg, PaCO2 65 mm Hg 3 pH 7.48, PaO2 79 mm Hg, PaCO2 43 mm Hg 4 pH 7.40, PaO2 65 mm Hg, PaCO2 38 mm Hg

2 RATIONALE: A markedly elevated PaCO2 is indicative of impending respiratory failure, requiring immediate medical attention. While the other ABGs are abnormal, the ABG requiring immediate intervention is that with the highest PaCO2 and associated acidosis.

While teaching a 45-year-old patient with asthma about the appropriate use of a peak flow meter, the nurse instructs the patient to notify the health care provider immediately if which situation occurs? 1 Wheezing is improved moderately with the use of a bronchodilator. 2 Less than 50% of the patient's personal best is achieved. 3 The short-acting bronchodilator is being used every three to four days. 4 Peak flow measurements remain unchanged after exercise.

2 RATIONALE: Achieving less than 50% of the patient's personal best on the peak flow meter indicates a medical emergency related to poor gas exchange and air flow. The patient should notify the health care provider immediately. Wheezing should be improved with a bronchodilator. Short acting bronchodilators used every one to two days indicate the need for additional asthma treatment. Peak flow measurements should not decrease following exercise if asthma is well-controlled.

When can airborne infection isolation for a patient with pulmonary tuberculosis (TB) be discontinued? 1. Once isoniazid drug therapy has been initiated 2. After three consecutive acid-fast bacillus (AFB) smears are negative 3. After effective instruction on the use of a high-efficiency particulate air (HEPA) mask 4. When two consecutive negative x-ray results are confirmed

2 RATIONALE: Airborne infection isolation is indicated for the patient with pulmonary or laryngeal TB until the patient is noninfectious (defined as effective drug therapy, clinical improvement, and three negative AFB smears). Therapy must be deemed effective. Teaching the patient to properly use the HEPA mask isn't criteria to terminate isolation. Chest x-rays do not meet criteria to terminate isolation.

After assisting the health care provider with a bedside thoracentesis, the nurse should continue to assess the patient for signs and symptoms of what? 1 Bronchospasm 2 Pneumothorax 3 Pulmonary edema 4 Respiratory acidosis

2 RATIONALE: Because thoracentesis involves the introduction of a catheter into the pleural space, there is a risk of pneumothorax. Thoracentesis does not carry a significant potential for causing bronchospasm, pulmonary edema, or respiratory acidosis.

hich strategy by the nurse would be most helpful in treating a patient with pneumonia and asthma who is experiencing chills? 1 Apply heat to the patient's posterior neck 2 Provide a blanket to the patient 3 Encourage the patient to bathe in hot water 4 Administer acetaminophen (Tylenol) 650 mg as prescribed

2 RATIONALE: Chills often occur in cycles and last for 10 to 30 minutes at a time. They usually signal the onset of an increase in temperature. For this reason, the nurse should provide a light blanket for comfort, but avoid overheating the patient with heat on the neck or bathing in hot water. Acetaminophen would be used if the patient became febrile following the chills.

A patient is complaining of a history of sinus infections off and on since childhood. The nurse knows that with chronic sinusitis: 1 Severe pain occurs 2 Diagnosis is difficult 3 Nasal drainage is purulent 4 Fever starts before the infection

2 RATIONALE: Chronic sinusitis is hard to diagnose because symptoms are often nonspecific. Severe pain and purulent nasal drainage do not occur. Patients are rarely febrile.

The nurse is assessing a patient who may have manifestations of chronic obstructive pulmonary disease (COPD). Which of these is a clinical manifestation of early COPD? 1 Dyspnea at rest 2 A chronic, intermittent cough 3 The presence of chest breathing 4 Production of copious amounts of sputum

2 RATIONALE: Clinical manifestations of COPD typically develop slowly. A chronic intermittent cough, which is often the first symptom to develop, later may be present every day as the disease progresses. Typically dyspnea is progressive, usually occurs with exertion, and is present every day. Dyspnea at rest and chest breathing are manifestations of late COPD. The cough may be unproductive of mucus.

The nurse is monitoring a patient who is having a thoracentesis for recurrent pleural effusion. Which of these assessment findings would be of most concern? 1. Removal of 1000 mL of pleural fluid 2. Restlessness and sudden complaint of dyspnea 3. SpO2 reading of 96% while on 2 L/minute of oxygen 4. Patient complaint of pressure at the needle insertion site

2 RATIONALE: During and after a thoracentesis, monitor the patient's vital signs and pulse oximetry and observe the patient for any manifestations of respiratory distress, which may indicate a possible complication, such as pneumothorax or pulmonary edema. It is not unusual to remove up to 1000 to 1200 mL of pleural fluid at one time. The SpO2 reading of 96% and patient complaint of pressure at the needle insertion site are not abnormal findings.

When teaching a patient about chronic obstructive pulmonary disease (COPD) rehabilitation, what strategy should the nurse teach the patient as essential to perform for energy conservation? 1. Complete inactivity 2. Exercise training 3. Reduced water intake 4. Reduced food intake

2 RATIONALE: Exercise training leads to energy conservation, which is an important component in COPD rehabilitation. Complete inactivity may alleviate symptoms acutely but is not helpful in the long term, because the patient needs to learn effective ways to improve muscle function. It is also important to reduce dyspnea by exercise training. Reduced water and food intake is not advisable; instead, increased water and food intake is essential to maintain energy and to loosen the secretions.

Which test result identifies that a patient with asthma is responding to treatment? 1.An increase in CO2 levels 2.A decreased exhaled nitric oxide 3.A decrease in white blood cell count 4.An increase in serum bicarbonate levels

2 RATIONALE: Nitric oxide levels are increased in the breath of people with asthma. A decrease in the exhaled nitric oxide concentration suggests that the treatment may be decreasing the lung inflammation associated with asthma and adherence to treatment. An increase in CO2 levels, decreased white blood cell count, and increased serum bicarbonate levels do not indicate a positive response to treatment in the asthma patient.

The nurse is assessing a patient with a diagnosis of restrictive lung disease caused by an extrapulmonary condition. Which finding indicates the patient may be suffering from a chest wall problem? 1 Brain tumor 2 Obesity hypoventilation 3 Paralysis of lower extremities 4 Recent changes in level of consciousness (LOC)

2 RATIONALE: Pickwickian syndrome is an obesity-ventilation condition of the chest wall that is a cause of extrapulmonary restrictive lung disease. A brain tumor and recent changes in LOC are related to a CNS cause. Paralysis of lower extremities is related to a neuromuscular system problem, and the patient may be suffering from Guillian-Barré Syndrome.

The patient had abdominal surgery yesterday. Today the lung sounds in the lower lobes have decreased. The nurse knows this could be because of what occurring? 1. Pain 2. Atelectasis 3. Pneumonia 4. Pleural effusion

2 RATIONALE: Postoperatively there is an increased risk for atelectasis from anesthesia as well as restricted breathing from pain. Without deep breathing to stretch the alveoli, surfactant secretion to hold the alveoli open is not promoted. Pneumonia will occur later after surgery. Pleural effusion occurs because of blockage of lymphatic drainage or an imbalance between intravascular and oncotic fluid pressures, which is not expected in this case.

What is the most appropriate nursing intervention for a patient learning about cetirizine (Zyrtec)? 1. Warn the patient to avoid driving or making important decisions 2. Prepare the patient to expect little to no side effects 3. Instruct the patient that urinary hesitancy is a common side effect 4. Explain to the patient that alcohol intake has an additive depressive effect

2 RATIONALE: Preparing the patient to expect little to no side effects is correct because the appropriate nursing intervention for a patient taking a second-generation antihistamine, such as cetirizine, includes informing the patient of the paucity of side effects when compared with first-generation antihistamines. Avoiding drinking or making important decisions, expecting urinary hesitancy, or explaining that alcohol has an additive depressive effect are teaching interventions applying to first generation antihistamines, not second-generation.

Which task can the registered nurse (RN) delegate to an unlicensed assistive personnel (UAP) in the care of a stable patient who has a tracheostomy? 1 Assessing the need for suctioning 2 Suctioning the patient's oropharynx 3 Assessing the patient's swallowing ability 4 Maintaining appropriate cuff inflation pressure

2 RATIONALE: Providing the individual has been trained in correct technique, UAP may suction the patient's oropharynx. Assessing the need for suctioning should be performed by an RN or licensed practical nurse, whereas swallowing assessment and the maintenance of cuff inflation pressure should be performed solely by the RN.

A patient presents to the emergency department with sudden-onset wheezing and coughing with progressive respiratory distress. What condition or diagnosis does the nurse recognize? 1. Acute bronchitis 2. An asthma attack 3. Pulmonary edema 4. Congestive heart failure

2 RATIONALE: Sudden onset of coughing and wheezing are the initial and most obvious symptoms of an acute asthma attack. An asthma attack may begin mild but progress to respiratory distress and arrest if it goes untreated. Acute bronchitis is irritation and inflammation of the mucous membrane lining of the respiratory tract, usually caused by an infectious agent. Pulmonary edema is fluid accumulation in the lungs due to heart failure or lung injury. The fluid collection impairs gas exchange and may result in respiratory failure. Congestive heart failure, or heart failure, is a condition in which the heart cannot pump effectively. Fluid may accumulate in the lungs. Edema may develop in the lower extremities, and shortness of breath may also occur with increasing frequency and severity.

The family of a patient with newly diagnosed tuberculosis is tested for infection with Mycobacterium tuberculosis. The patient's wife, who has a history of alcoholism, has had two negative Mantoux (PPD) tests. Both of their children have positive Mantoux results. The nurse recognizes that a course of preventive treatment with isoniazid will be required for which family member(s)? 1. The spouse only 2. Both children only 3. The spouse and the children 4. Neither the spouse nor the children

2 RATIONALE: The children would be given isoniazid (INH) because of exposure and conversion from a negative to a positive Mantoux result. The wife would not be treated because she has not converted to positive and may be susceptible to INH hepatitis, a susceptibility that increases with age. She may also be susceptible to hepatotoxicity because of her age and history of alcoholism. The children definitely require preventive treatment with INH.

The nurse caring for a patient with respiratory failure finds that the patient has hypoxemia due to ventilation-perfusion (V/Q) mismatch. What is the primary intervention that the nurse should implement in this case? 1 Administer antibiotics as ordered. 2 Give oxygen therapy as ordered. 3 Administer bronchodilators as ordered. 4 Give antisecretary agents as ordered

2 RATIONALE: The first intervention for any hypoxemia patient is to administer oxygen. Oxygen therapy can reverse hypoxemia caused by V/Q mismatch, because not all gas exchange units are affected. Antibiotics improve V/Q mismatch only in infective conditions of the lung. Bronchodilators improve V/Q mismatch only if bronchospasm is the underlying cause. Anti-secretary agents improve V/Q mismatch only if increased bronchial secretions are the underlying cause.

Before discharge, the nurse discusses nutrition with the patient with emphysema and pneumonia. The nurse instructs the patient to: 1. Eat three large meals per day 2. Rest for 30 minutes before eating 3. Increase intake of hot foods 4. Exercise before meals to increase appetite

2 RATIONALE: The patient with emphysema should conserve energy to eat and should rest for at least 30 minutes before eating to increase energy needed to eat. The patient should consume five to six small meals per day, avoid hot foods, and exercise after eating to conserve energy.

The purpose of this exercise is to prolong exhalation and thereby prevent bronchiolar collapse and air trapping. 1 Diaphragmatic breathing 2 Pursed-lip breathing (PLB) 3 Huff coughing 4 Chest physiotherapy (CPT)

2 RATIONALE: The purpose of PLB is to prolong exhalation and thereby prevent bronchiolar collapse and air trapping. PLB is simple and easy to teach and learn, and it gives the patient more control over breathing, especially during exercise and periods of dyspnea. Another type of breathing retraining exercise is diaphragmatic breathing, which focuses on using the diaphragm to achieve maximum inhalation and slow respiratory rate, not prolong exhalation. Huff coughing is an effective forced expiratory technique, not a breathing exercise to prolong exhalation. Chest physiotherapy (CPT) consists of postural drainage, percussion, and vibration and is for patients who have difficulty clearing excessive bronchial secretions.

A patient has been prescribed topical decongestants for sinusitis. What information should the nurse include when teaching the patient about this medication? 1 Use these medications for at least 10 days. 2 Use these medications for a maximum of 3 days. 3 Use these medications for at least 1 month. 4 Use these medications for at least a week

2 RATIONALE: Topical decongestants should be used for no more than 3 days to avoid rebound congestion by vasodilatation. If used longer than 3 days, the patient's symptoms may be made worse.

A nurse finds that the tracheostomy tube of a patient is dislodged. The nurse is unable to replace the tube and calls for assistance. Until assistance arrives, the nurse should perform what intervention? 1. Cover the stoma with sterile gauze. 2. Put the patient in a semi-Fowler position to relieve dyspnea. 3. Ventilate the patient with a bag-mask. 4. Remove the retention sutures.

2 RATIONALE: When the dislodged tracheostomy tube cannot be replaced single-handedly, the nurse should put the patient in the semi-Fowler position. This position helps to prevent dyspnea until assistance arrives. The tracheostomy stoma should be covered only if a bag-mask ventilation needs to be performed, which is done only in case of respiratory arrest. The retention sutures help to reinsert the tube and should not be removed.

The health care provider has prescribed intravenous (IV) vancomycin for a patient with pneumonia. Which action should the nurse perform first? 1. Obtain a full set of vital signs 2. Obtain sputum cultures for sensitivity 3. Draw a blood specimen to evaluate the white blood cell count 4. Administer the antibiotic over at least 60 minutes

2 RATIONALE: The nurse should ensure that the sputum for culture and sensitivity has been sent to the laboratory before administering the antibiotic. It is important that the organisms be correctly identified (in the culture) before their numbers are affected by the antibiotic; the test also will determine whether the proper antibiotic has been prescribed (sensitivity testing). Vital signs and white blood cell count measurement can be assessed following the obtainment of sputum cultures. Timing of antibiotic administration should be based upon the institution's policy.

A patient with an acute attack of asthma is in a state of panic. Which nursing measures help to relieve the panic? Select all that apply. 1. Use sedation. 2. Encourage pursed-lip breathing. 3. Utilize the "talking down" technique. 4. Utilize a "walking down" technique. 5. Be calm, quiet, and reassuring

2,3,5 RATIONALE: Pursed-lip breathing keeps the airways open, slows down the respiratory rate, and encourages deep breathing. "Talking down" is a technique that helps to calm the patient. A calm, quiet, and reassuring nurse helps to pacify the patient. Use of sedatives should not be encouraged as they may cause respiratory depression. There is no technique called "walking down."

A patient presents with epistaxis. The primary health care provider places an anterior packing in the form of a compressed sponge to control the bleed. Which nursing interventions would be appropriate for this patient? Select all that apply. 1. Provide extra packing material. 2. Administer mild opioid medication. 3. Administer prophylactic antibiotics. 4. Provide saline nasal spray. 5. Administer nonsteroidal antiinflammatory drugs

2,3 RATIONALE: Nasal packing is painful because of the pressure applied to stop the bleeding. Mild opioid medication should be given to the patient for analgesia. Patients with nasal packing are susceptible to staphylococci infection, so prophylactic antibiotics should be administered. Extra packing material is no longer required as anterior packing is already done. Saline nasal spray is recommended after packing removal. Nonsteroidal antiinflammatory drugs are discouraged due to the risk of bleeding.

A patient presents to the emergency department with acute exacerbation of asthma. What actions should the nurse perform to monitor the patient's respiratory and cardiovascular systems? Select all that apply. 1. Take a chest radiograph. 2. Auscultate the lung sounds. 3. Measure blood pressure and respiratory rate. 4. Monitor arterial blood gases (ABGs) and pulse oximetry. 5 Check the patient's temperature

2,3,4 RATIONALE: It is essential to monitor respiratory and cardiovascular systems in case of acute exacerbation of asthma. Auscultating lung sounds, measuring blood pressure and respiratory rate, and monitoring ABGs and pulse oximetry are required to monitor these systems. Chest radiographs are seldom useful in the management of an acute asthma attack. Checking the temperature may not contribute to monitoring respiratory and cardiovascular systems.

A nurse is caring for a patient with bronchitis. When reviewing the medical records, the nurse finds that the patient has decreased lung compliance. What are the conditions in which lung compliance decreases? Select all that apply. 1 Chronic obstructive pulmonary disease (COPD) 2 Pneumonia 3 Pulmonary edema 4 Pleural effusion 5 Sarcoidosis

2,3,4,5 RATIONALE: Lung compliance is a measurement of the ease with which the lungs expand. It decreases in cases of pneumonia, pulmonary edema, and pleural effusion because fluid builds up in the lungs. Lung compliance also decreases when lung tissue becomes less elastic, as in sarcoidosis. In COPD, lung compliance increases, since there is destruction of the alveolar walls and loss of tissue elasticity.

The laryngoscopy report of a patient indicates the presence of a cancerous mass over the vocal cords. The patient is to be treated with radiation therapy. How can a nurse help the patient cope with the side effects of radiation therapy? Select all that apply. 1 Encourage alcohol consumption. 2 Encourage increased fluid intake. 3 Administer pilocarpine hydrochloride. 4 Discourage use of artificial saliva. 5 Encourage intake of soft food.

2,3,5 RATIONALE: Dry mouth is the most common side effect of radiation therapy. The nurse should instruct the patient to increase fluid intake, as hydration will help the patient to relieve symptoms. Due to radiation therapy, the patient's saliva decreases in volume and becomes thick. Pilocarpine hydrochloride is helpful in increasing saliva production. Soft and bland food causes less irritation to the oral mucosa and should be encouraged if the patient has oral mucositis. Alcohol cessation should be encouraged following the diagnosis of cancer to avoid complications. Artificial saliva also helps in keeping the mouth hydrated in xerostomia.

The nurse is monitoring a patient who has pneumonia with thick secretions. The patient is having difficulty clearing the secretions. Which of these would be appropriate nursing interventions for this patient? Select all that apply. 1. Perform postural drainage every hour. 2. Encourage the patient to rest and limit activity. 3. Provide adequate hydration by encouraging fluid intake. 4. Provide analgesics as prescribed to promote patient comfort. 5. Teach the patient how to cough effectively to bring secretions to the mouth.

2,3,5 RATIONALE: nterventions for pneumonia include teaching the patient how to cough effectively to remove secretions, providing adequate hydration, and encouraging rest. Hydration is important in the supportive treatment of pneumonia to prevent dehydration and loosen secretions. Individualize and carefully monitor fluid intake if the patient has heart failure. It is not necessary to provide postural drainage every hour. Providing analgesics will not help the patient clear secretions.

A patient with pneumonia is being treated at home and has reported fatigue to the nurse. What instructions should the nurse include when teaching the patient about care and recovery at home? Select all that apply. 1 Resume work to build strength. 2 Get adequate rest. 3 Restrict fluid intake. 4 Avoid alcohol and smoking. 5 Take every dose of the prescribed antibiotic.

2,4,5 RATIONALE: To ensure complete recovery after pneumonia, the patient should be advised to rest, avoid alcohol and smoking, and take every dose of the prescribed antibiotic. The patient should not resume work if feeling fatigued and should be encouraged to drink plenty of fluids during the recovery period.

When teaching a patient about using a dry powder inhaler, what instructions should be included in this teaching? Select all that apply. 1. Shake the inhaler well before use. 2. Avoid shaking the inhaler before use. 3. Slow down the inspiration. 4.Increase the inspiration. 5. Keep the device moisture-free

2,4,5 RATIONALE: With dry powder inhalers, there is no need to shake before use. Inspiration should be rapid, and the device should be kept moisture-free to protect the dry powder. Shaking the inhaler well before use and slow inspiration are actions performed when using metered-dose inhalers.

A patient has been prescribed inhalational steroids. What information should the nurse include when explaining to the patient about inhalational steroids? Select all that apply. 1 They are used for acute respiratory infections. 2 They are not used for acute respiratory failure. 3 Prolonged use can cause adrenal insufficiency. 4 They exacerbate hypokalemia caused by diuretics. 5 They require 4 to 5 days for optimum therapeutic effects.

2,5 RATIONALE: Inhaled steroids are not used for acute respiratory failure because of delayed onset of action. Inhaled steroids require 4 to 5 days for optimum therapeutic effects. These are not used for respiratory infections, as they do not act on pathogenic organisms. Inhaled steroids do not cause adrenal insufficiency; prolonged intravenous (IV) use, however, can cause adrenal insufficiency. IV use of corticosteroids exacerbates hypokalemia caused by diuretics.

The patient has a prescription for albuterol 5 mg via nebulizer. Available is a solution containing 2 mg/mL. Calculate how many milliliters the nurse should use to prepare the patient's dose. Fill in the blank. __mL

2.5 RATIONALE: 5 mg ÷ 2 mg/mL = 2.5 mL

The nurse is caring for an adult patient with bronchiectasis. The nurse knows that the primary cause of this disease in adults is related to: 1. Adult-onset asthma 2. Heavy smoking for more than 20 years 3. Untreated or delayed treatment of bacterial lung infections 4. Genetic predisposition and family members with the same diagnosis

3 RATIONALE: Bronchiectasis is an obstructive lung disease, generally caused by bacterial lung infections that are either not treated or treatment is delayed, leading to inflammation, airway damage, and remodeling. It can even follow a single severe bacterial infection if treatment is not given or delayed. Adult-onset asthma with no infection has no specific relationship to acquiring this disease. A patient's smoking history or genetically related factors do not affect the cause of this condition.

A patient with a spontaneous pneumothorax has a chest tube that is attached to a closed drainage system. The drainage unit is not attached to suction. The water level in the water-seal chamber is fluctuating. What action should the nurse take? 1 Notifying the primary health care provider immediately 2 Decreasing the amount of water in the water-seal chamber 3 Continuing to monitor and document the respiratory status 4 Clamping the chest tube as close as possible to the insertion site

3 In a closed drainage system (not in a suction system) the fluid in the water-seal chamber rises when the patient inhales and falls when the patient exhales; this is a normal finding. The absence of fluctuations may indicate an obstruction in the system. The nurse must continually check the function of the closed drainage system and assess respiratory status at least every 4 hours. There is no need to notify the primary healthcare provider or decrease the amount of water in the water-seal chamber, because the chest tube system is functioning normally. The chest tube should not be clamped unless the nurse is directed to do so by the primary healthcare provider, because as doing so could cause a tension pneumothorax. Text Reference - p. 545

The patient is hospitalized with pneumonia. Which diagnostic test should be used to measure the efficiency of gas transfer in the lung and tissue oxygenation? 1 Thoracentesis 2 Bronchoscopy 3 Arterial blood gases 4 Pulmonary function tests

3 Arterial blood gases are used to assess the efficiency of gas transfer in the lung and tissue oxygenation, as is pulse oximetry. Thoracentesis is used to obtain specimens for diagnostic evaluation, remove pleural fluid, or instill medication into the pleural space. Bronchoscopy is used for diagnostic purposes, to obtain biopsy specimens, and to assess changes resulting from treatment. Pulmonary function tests measure lung volumes and airflow to diagnose pulmonary disease, monitor disease progression, evaluate disability, and evaluate response to bronchodilators. Text Reference - p. 491

A nurse is teaching a patient how to manage fatigue induced by radiation therapy. What statement by the patient indicates a correct understanding of the lesson? 1. "I will walk three to four hours every day to increase my level of energy." 2. "It's most important for me to avoid asking for help so I can become more independent." 3. "Because I have the most energy in the morning, I will plan my errands during this time." 4. "I will keep myself busy continuously throughout the daylight hours."

3 RATIONALE: "Because I have the most energy in the morning, I will plan my errands during this time" indicates that the patient understands the importance of doing activities that are most important to them and to rest during periods of low energy. Fatigue is a common side effect of radiation therapy and usually begins a few weeks into therapy. "I will walk three to four hours every day to increase my level of energy" indicates that the patient does not understand that scheduling activities for a period of three to four hours is excessive and does not allow time for adequate rest periods. It is important that patients suffering from radiation-induced fatigue identify support systems as a means of assistance. Avoiding requests for help would be counter to this teaching. Continuous engagement in activity would not provide periods of much needed rest for a patient who is fatigued from radiation therapy.

A patient with a history of epilepsy is admitted to the hospital for treatment of fever and shortness of breath. The patient is diagnosed with pneumonia. On taking history, the nurse finds that the patient had an episode of seizures four days ago with profuse vomiting. What type of pneumonia does the patient have? 1. Hospital-associated pneumonia 2. Community-acquired pneumonia 3. Aspiration pneumonia 4. Opportunistic pneumonia

3 RATIONALE: A patient who has seizures is at risk of developing aspiration pneumonia. The gastric contents enter the respiratory tract during the seizure and damage the lung tissue. Therefore this is the most probable reason for the patient's symptoms. The history of the patient does not suggest any exposure to pneumonia in the community. The patient has never been in the hospital; therefore hospital-associated pneumonia is highly unlikely. The patient does not have a history of HIV, intake of immunosuppressive drugs, corticosteroids, or any disorders leading to immunosuppression. Therefore opportunistic pneumonia did not occur in this patient.

A patient experiencing severe wheezing arrives in the emergency department and is diagnosed with severe exacerbation of asthma. During the admission assessment, the nurse on the inpatient unit notes that the patient continues to struggle with breathing, however, there is an absence of wheezing. How should the nurse interpret the assessment findings? 1. The patient has improved as there is no wheezing. 2. The patient is hypoxic and needs oxygen therapy. 3. The patient has respiratory failure and needs mechanical ventilation. 4. The patient has retained secretions and needs chest physiotherapy.

3 RATIONALE: A silent chest or absence of wheezing in a patient who had been having severe wheezing indicates an impending respiratory failure. The patient may need mechanical ventilation to support respiration. It is a sign of severe obstruction and it is a life-threatening condition. It is not a sign of improvement. Oxygen therapy may not help the patient as there is an obstruction in the airway. Chest physiotherapy helps in removing secretions from the airway, but may not be helpful in patients who are at risk of respiratory failure.

The nurse is caring for a patient with complaints of allergic nasal rhinitis. The nurse knows that the treatment includes: 1. Antibiotics 2. First-generation antihistamines 3.Adequate fluid intake when taking antihistamines 4. A two-drug combination with an antihistamine and corticosteroid spray

3 RATIONALE: Adequate fluid intake is essential when the patient is taking antihistamines. Antibiotics are not appropriate for allergic rhinitis. Second-generation antihistamines are preferred over first-generation because of their nonsedating effects. Initially monotherapy is prescribed, and if that is not effective then a two-drug combination may help.

The nurse evaluates that nursing interventions to promote airway clearance in a patient admitted with chronic obstructive pulmonary disease (COPD) are successful based on which finding? 1. Absence of dyspnea 2. Improved mental status 3. Effective and productive coughing 4. PaO2 within normal range for the patient

3 RATIONALE: Airway clearance is evaluated most directly as successful if the patient can engage in effective and productive coughing. Absence of dyspnea, improved mental status, and PaO2 within normal range for the patient show improved respiratory status but do not evaluate airway clearance.

The nurse determines that additional discharge teaching is needed for a patient with pneumonia when the patient states: 1 "I will seek medical attention if I develop a fever or productive cough." 2 "Breathing exercises may help prevent future infections." 3 "I should take antibiotics for all upper respiratory infections." 4 "I will take all medications as prescribed.

3 RATIONALE: Antibiotics are not indicated for all upper respiratory tract infections. It is important for the patient to continue with coughing and deep-breathing exercises for at least six weeks, until all of the infection has cleared from the lungs. The patient should take all medications as prescribed and seek medical attention for signs or symptoms of new infection.

The patient has been diagnosed with head and neck cancer. Along with the treatment for the cancer, what other treatment should the nurse expect? 1 Nasal packing 2 Epistaxis balloon 3 Gastrostomy tube 4 Peripheral skin care

3 RATIONALE: Because 50% of patients with head and neck cancer are malnourished before treatment begins, many patients need enteral feeding via a gastrostomy tube because the effects of treatment make it difficult to take in enough nutrients orally, whether surgery, chemotherapy, or radiation are used. Nasal packing could be used with epistaxis or with nasal or sinus problems. Peripheral skin care would not be expected because it is not related to head and neck cancer.

A patient with acute viral rhinitis has asked the nurse about medications that can be taken to relieve symptoms. Which statement, by the patient, reflects a need for further instruction? 1. "Antibiotics won't help this cold to go away." 2. "I can use saline nose spray to relieve congestion." 3. "I will limit the usage of nasal decongestant spray to 10 days." 4. "I will not drive if I take an antihistamine as it may make me drowsy.

3 RATIONALE: Decongestant sprays should not be used for more than three days to prevent rebound nasal congestion. Antibiotics may not be effective if the patient still has the cold. Saline nasal spray can be used to relieve congestion. The patient should not drive if taking an antihistamine.

A patient suspected of having lung cancer has undergone lung biopsy through transthoracic needle aspiration (TTNA). What is the priority nursing action for this patient immediately following the procedure? 1. Allow the patient to take a rest. 2. Instruct the patient to do deep breathing. 3. Send the patient for a chest x-ray as prescribed. 4. Measure oxygen saturation levels

3 RATIONALE: Following a lung biopsy through TTNA, the patient should be sent for chest x-ray to rule out a pneumothorax, which is a common complication of the procedure. Only after the chest x-ray is done, can the patient can be told to rest or do deep breathing exercises. Oxygen saturation levels are generally monitored throughout the procedure.

The nurse is caring for a patient suspected of having pneumonia. What instructions should the nurse provide to the patient prior to receiving a chest x-ray? 1. Instruct the patient to undress completely and put on a gown. 2. Instruct the patient to sign a consent form. 3. Instruct the patient to remove any metal between neck and waist. 4. Tell the patient to not have food 2 hours before the test.

3 RATIONALE: For the chest x-ray, the nurse should instruct the patient to remove any metal between neck and waist. The patient is not required to undress completely; as it is a chest x-ray, undressing to waist is sufficient. The test does not require the patient to sign a consent form, and there is no need to avoid food before the test.

What is the arterial oxygen/fraction of inspired oxygen (PaO2/FIO2, or P/F) ratio in acute respiratory distress syndrome (ARDS)? 1 Greater than 400 2 Greater than 300 3 Less than 200 4 Between 200 and 300

3 RATIONALE: In acute respiratory distress syndrome, the P/F ratio is less than 200. Under normal circumstances, the P/F ratio would be greater than 400 (e.g., 95/0.21 = 452). With the onset and progression of lung injury and impairment in oxygen delivery through the alveolar-capillary interface, the PaO2 may remain lower than expected despite increased FIO2. The P/F ratio is 200 to 300 in acute lung injury.

The nurse is aware of the value of using a mini-tracheostomy to facilitate suctioning when patients are unable to independently mobilize their secretions. For which patient is the use of a mini-tracheostomy indicated? 1. A patient whose recent ischemic stroke has resulted in the loss of the gag reflex 2. A patient who requires long-term mechanical ventilation as the result of a spinal cord injury 3. A patient whose increased secretions are the result of community-acquired pneumonia 4. A patient with a head injury who has developed aspiration pneumonia after the patient's family insisted on spoon-feeding the patient

3 RATIONALE: It is appropriate to suction a patient with pneumonia using a mini-tracheostomy if blind suctioning is ineffective or difficult. An absent or compromised gag reflex, long-term mechanical ventilation, and a history of aspiration contraindicates the use of a mini-tracheostomy.

A patient with allergic asthma has been prescribed omalizumab (Xolair). The patient requests that the medication be administered at home for convenience. Which nursing action is appropriate in this case? 1 Load the injection and provide it to the patient for self-administration. 2 Administer the injection to the patient on a home visit. 3 Explain to the patient that the medication should be administered strictly at the clinic. 4 Ask a family member to administer the medication at home.

3 RATIONALE: Omalizumab (Xolair) is a monoclonal antibody to IgE that decreases circulating free IgE levels. The drug prevents IgE from attaching to mast cells, preventing the release of chemical mediators that may exacerbate asthma. The medication can cause anaphylactic reaction and should be administered at a clinic that is well-equipped to handle emergencies. The nurse should not encourage self-administration of the medication at home, as the anaphylactic reaction can be life-threatening. The nurse may not be well equipped to handle emergencies at home, so the medication should not be administered during a home visit. A family member should not be asked to administer the medication for the same reason.

A nurse is caring for a patient diagnosed with acute respiratory distress syndrome. The nurse is aware that these patients often will require which intervention? 1 Peritoneal dialysis 2 Frequent suctioning 3 Mechanical ventilation 4 Creatinine and blood urea nitrogen (BUN) testing

3 RATIONALE: Patients with acute respiratory distress syndrome likely will require mechanical ventilation to support their respiratory status. Frequent suctioning is not required often, but some suctioning may be required. Peritoneal dialysis and creatinine and BUN testing might be necessary with some level of kidney failure, not respiratory compromise.

A patient with chronic obstructive pulmonary disease (COPD) experiences dyspnea and has a forced expiratory volume (FEV1) of 70% of predicted value. The nurse expects that what will be included on the patient's treatment plan? 1. Use of inhalational budesonide (Pulmicort Flexhaler) 2. Use of long-acting theophylline (Theo-24) 3. Use of short-acting bronchodilator 4. Using a combination of fluticasone and salmeterol (Advair Diskus)

3 RATIONALE: Since the patient has FEV1 of 70% and is suffering from mild COPD, the patient may benefit from the use of short-acting bronchodilators. Budesonide, being a steroid, is not used for treating mild COPD. Use of theophylline in COPD is controversial and should be used only in patients who do not respond to other drugs. A combination of fluticasone and salmeterol is not required for treating mild COPD; this medication is prescribed to patients who have a FEV1 of less than 60%.

The nurse is assigned to care for a patient in the emergency department admitted with an exacerbation of asthma. The patient has received a β-adrenergic bronchodilator and supplemental oxygen. If the patient's condition does not improve, the nurse should anticipate what as the most likely next step in treatment? 1. Intravenous (IV) fluids 2.Biofeedback therapy 3.Systemic corticosteroids 4.Pulmonary function testing

3 RATIONALE: Systemic corticosteroids speed the resolution of asthma exacerbations and are indicated if the initial response to the β-adrenergic bronchodilator is insufficient. IV fluids may be used, but not to improve ventilation. Biofeedback therapy and pulmonary function testing may be used after recovery to assist the patient and monitor the asthma.

An asthmatic patient was prescribed theophylline (Theo-24). A nurse understands that the patient is at risk for tachycardia and seizures. In regards to safety, the nurse expects that what will be included on the patient's treatment plan? 1. Encourage the use of caffeine. 2. Monitor serum blood levels of adrenaline. 3. Monitor serum blood levels of theophylline. 4. Use diazepam (Valium) to prevent seizures.

3 RATIONALE: Tachycardia and seizures are known toxic effects of theophylline at higher blood levels. In addition, the drug has a narrow margin of safety. Therefore, monitoring blood levels of theophylline helps to reduce such toxic effects. Caffeine increases the toxic effects of theophylline. Monitoring plasma levels of adrenaline may not help as tachycardia is not associated with adrenaline in this case. Prophylactic use of diazepam may not prevent seizures as they are caused by the use of theophylline.

To promote airway clearance in a patient with pneumonia and asthma, the nurse instructs the patient to perform which action? 1. Wear supplemental oxygen at all times 2. Perform pursed-lip breathing 3. Sit upright while using the flutter device 4. Use the incentive spirometer 10 times per hour

3 RATIONALE: The flutter device is used to increase mucus production to promote airway clearance and gas exchange; it should be used while the patient is in an upright position. Supplemental oxygen may not be indicated depending on the oxygen saturation level. Pursed-lip breathing and the incentive spirometer will not promote airway clearance.

A patient has been admitted with a suspected lung abscess. During the assessment, the nurse is aware that the most common manifestation of a lung abscess is which of these? 1 Fever 2 Vomiting 3 Purulent sputum that has a foul odor and taste 4 Increased breath sounds on auscultation over the involved segment of lung.

3 RATIONALE: The most common manifestation of a lung abscess is cough-producing purulent sputum (often dark brown) that is foul smelling and foul tasting. Hemoptysis is common, especially when an abscess ruptures into a bronchus. Other common manifestations are fever, chills, prostration, night sweats, pleuritic pain, dyspnea, anorexia, and weight loss. Physical examination of the lungs indicates dullness to percussion and decreased breath sounds on auscultation over the involved segment of lung. Vomiting is not a manifestation of a lung abscess.

A patient who was in a motor vehicle accident is brought to the emergency department unconscious, and cardiopulmonary resuscitation (CPR) is performed. The patient responds well, and the condition improves. After several hours, the patient experiences dyspnea and becomes cyanotic. On examination the neck veins are distended, and the patient is tachycardic. The nurse expects that the immediate plan for treatment will include what intervention? 1. Oxygen administration 2. Pericardiocentesis 3. Needle decompression 4. Placing the patient in a side-lying position

3 RATIONALE: The symptoms and signs indicate that the patient has tension pneumothorax. This is a medical emergency in which air enters into the pleural space and does not come out. This leads to compression of the surrounding organs like the lung, heart, and large vessels. If not treated promptly, the patient may die. Therefore the patient requires immediate needle decompression followed by chest tube insertion with chest drainage system. Oxygen administration will not help this patient. Pericardiocentesis is done for patients with cardiac tamponade. Repositioning the patient to his side is not required in this case.

A patient has a chest tube inserted to treat a spontaneous pneumothorax. Which observation causes the nurse to conclude that the water-seal chamber of the closed chest drainage system is functioning properly? 1. Gentle bubbling in the suction chamber 2. Patient tolerating mild shortness of breath 3. Water-seal chamber level fluctuating with respirations 4. The presence of bloody drainage in the water-seal chamber

3 RATIONALE: The water-seal chamber level fluctuates with respirations as a result of the restoration of negative pressure within the thoracic cavity. Gentle bubbling in the suction chamber indicates a possible air leak. New-onset of shortness of breath in a patient with a chest tube requires further assessment. The water-seal chamber should not contain blood; this finding indicates that the chest tube drainage system may have been knocked onto its side and should be replaced.

What nursing intervention is most appropriate to enhance oxygenation in a patient with unilateral malignant lung disease? 1.Positioning patient on right side 2. Maintaining adequate fluid intake 3. Positioning patient with "good lung" down 4. Performing postural drainage every four hours

3 RATIONALE: Therapeutic positioning identifies the best position for the patient, thus assuring stable oxygenation status. Research indicates that positioning the patient with the unaffected lung (good lung) dependent best promotes oxygenation in patients with unilateral lung disease. For bilateral lung disease, the right lung down has best ventilation and perfusion. Increasing fluid intake and performing postural drainage will facilitate airway clearance, but positioning is most appropriate to enhance oxygenation.

The nurse is caring for an alert patient with a diagnosis of recent nasal fracture. The nurse knows that initially the best way to maintain the airway is to: 1. Insert a nasal airway 2. Apply oxygen via a face mask 3. Apply ice to face and nose per protocol 4. Place patient in a semi-Fowler position

3 RATIONALE: Within the first 48 hours of the nasal fracture, ice needs to be applied to the face and nose in 10 to 20 minute intervals to help reduce edema and bleeding. Nasal airways should not be inserted into patients with a nasal fracture. In addition, airways should not be inserted into alert patients. Patients need to be placed in an upright position and not a semi-Fowler position. Oxygen via a face mask is not needed to keep the airway open.

A nurse is caring for a patient who has undergone laryngectomy. Upon suctioning the tracheostomy tube, the nurse notices a slight thickening of the secretions. Which measures should the nurse take to prevent thickening of secretions? Select all that apply. 1 Keep the patient in a semi-Fowler position. 2 Change the tracheostomy tube. 3 Maintain adequate fluid intake. 4 Humidify inspired gases. 5 Give a normal saline bolus through tracheostomy

3,4 RATIONALE: Maintaining adequate hydration through intravenous or enteral route and humidifying the inspired gases prevents drying and thickening of secretions. This allows easy suction of secretions. Changing the patient's position helps to relieve dyspnea in the patient. Changing the tracheostomy tube does not help as the secretions are too copious to be removed. Normal saline bolus is not recommended as it may cause infection

The nurse is caring for a patient on mechanical ventilation. What are the nursing interventions that prevent the development of volutrauma in a patient on a ventilator? Select all that apply. 1 Sterile techniques 2 Strict hand washing 3 Smaller tidal volumes 4 Pressure-control ventilation 5 Mouth care and oral hygiene

3,4 RATIONALE: Volutrauma occurs when large tidal volumes are given to a mechanically ventilated patient. Because of the high tidal volume, the alveoli may become damaged and tear, allowing proteins and fluid to move into the alveolar spaces. This can be prevented by giving smaller tidal volumes or pressure-control ventilation. Strict hand washing, sterile technique during endotracheal suctioning, and frequent mouth care and oral hygiene are helpful to prevent ventilator-associated pneumonia and not volutrauma.

The nurse is caring for the patient with chronic obstructive pulmonary disease who is undernourished and underweight. Which steps can the nurse take to improve the patient's nutritional status? Select all that apply. 1. Restrict fluid intake to 1 L/day 2. Have the patient drink fluid with meals 3. Provide a diet high in protein and calories 4. Provide five to six small meals per day 5. Avoid overfeeding the patient

3,4,5 RATIONALE: A diet high in calories and protein, moderate in carbohydrates, and moderate to high in fat is recommended and can be divided into five or six small meals a day. High-protein, high-calorie nutritional supplements can be offered between meals. Nonprotein calories should be divided evenly between fat and carbohydrate, but avoid overfeeding the patient. Fluid intake should be at least 3 L/day unless contraindicated by other medical conditions. Fluids should be taken between meals (rather than with them) to prevent excess stomach distention and to decrease pressure on the diaphragm.

The nurse is caring for a patient who had a recent lung transplant. The nurse knows which of the following? Select all that apply. 1 Acute rejection can occur 2 to 3 weeks after surgery. 2 Immunosupressive therapy is usually a two-drug regimen. 3 Accurate diagnosis of rejection is by transtracheal biopsy. 4 Cytomegalovirus (CMV) pneumonia is the most common opportunistic infection. 5 During the first year, viral pneumonia is the most common postoperative infection. 6 Lung transplant recipients usually receive higher levels of immunosuppressive therapy than other organ recipients.

3,4,6 RATIONALE: Cytomegalovirus (CMV) is the most common opportunistic infection. Because acute rejection is common, higher levels of immunosuppressive therapy than what other organ recipients receive are given. Diagnosis of rejection is confirmed by transtracheal biopsy. Immunosuppressive therapy is usually a three-drug regimen and acute rejection can occur in the first 5 to 10 postoperative days. If acute rejection is diagnosed, then high doses of corticosteroids are given for three days, followed by high doses of oral prednisone. If this does not work and rejection occurs, then antilymphocytic therapy may be useful. Bacterial pneumonia is the most common postoperative infection.

The health care provider has prescribed salmeterol (Serevent) for a patient with asthma. In reviewing the use of dry powder inhalers (DPIs) with the patient, what instructions should the nurse provide? 1 "Close lips tightly around the mouthpiece and breathe in deeply and quickly." 2 "To administer a DPI, you must use a spacer that holds the medicine so that you can inhale it." 3 "You will know you have correctly used the DPI when you taste or sense the medicine going into your lungs." 4 "Hold the inhaler several inches in front of your mouth and breathe in slowly, holding the medicine as long as possible."

4 The patient should be instructed to tightly close the lips around the mouthpiece and breathe in deeply and quickly to ensure the medicine moves down deeply into the lungs. Dry powder inhalers do not require spacer devices. The patient may not taste or sense the medicine going into the lungs. The inhaler should not be placed several inches in front of the mouth.

Which factor places a conscious patient at risk for pneumonia? 1 Effective postoperative pain management 2 Lying supine for two consecutive hours 3 Adequate cough and deep breathing exercises 4 Difficulty swallowing medication

4 The patient who has difficulty swallowing needs assistance in eating, drinking, and taking medication to prevent aspiration. Difficulty swallowing increases risk of aspiration. Treating postoperative pain effectively provides comfort, permitting the patient to cough and deep breathe and achieve optimum mobility. Lying supine for two consecutive hours alone does not place an otherwise healthy patient at risk for pneumonia but for the altered consciousness patient, repositioning should occur at least every two hours. Turning, coughing, and deep breathing exercises promote optimal oxygenation or perfusion and help prevent atelectasis. Text Reference - p. 547

The nurse is assessing a young male patient who came to the emergency department complaining of sudden shortness of breath. He has no other visible problems. The nurse notes that, upon auscultation, there are no breath sounds on the right upper lobe of the lung. The nurse suspects that the patient has which of these conditions? 1 Tension pneumothorax 2 Iatrogenic pneumothorax 3 Traumatic pneumothorax 4 Spontaneous pneumothorax

4 RATIONALE: A lack of breath sounds over a portion of the lung fields indicates the presence of a pneumothorax. A spontaneous pneumothorax typically occurs because of the rupture of small blebs (air-filled blisters) located on the apex of the lung. These blebs can occur in healthy, young individuals, especially tall, thin males. Tension pneumothorax occurs when air enters the pleural space but cannot escape. The continued accumulation of air in the pleural space causes increasingly elevated intrapleural pressures. Tension pneumothorax can occur with mechanical ventilation and resuscitative efforts. Iatrogenic pneumothorax can occur because of laceration or puncture of the lung during medical procedures. Traumatic pneumothorax can occur from either penetrating (open) or nonpenetrating (closed) chest trauma.

The nurse provides care to a trauma victim. Which clinical manifestation most suggests a pneumothorax? 1. Inspiratory crackles 2. Pronounced crackles 3. Dullness on percussion 4. Absence of breath sound

4 RATIONALE: A pneumothorax indicates that one of the lungs has collapsed and is not functioning. On auscultation no sounds of air movement will be heard. Because no air movement occurs with a pneumothorax, no breath sounds, including crackles, will be heard. Dullness may be a finding on percussion over the area of the pneumothorax, but an absence of breath sounds is the definitive finding.

The nurse is caring for a patient with a diagnosis of active tuberculosis (TB). The nurse knows that: 1. Directly observed therapy is used only in the initial phase 2. Drug therapy is in three phases (initial, interim, and continuation) 3. Liver function tests (LFTs) are initiated 14 days after the start of treatment 4. Alcohol is avoided because it increases the hepatotoxicity of isoniazid (INH)

4 RATIONALE: Alcohol must be avoided because it increases the hepatotoxicity of INH. Directly observed therapy must be continued through both phases in patients who are at risk for non-adherence to drug therapy. Drug therapy includes a two-phase process, with an initial and continuation phase. Baseline LFTs are done before treatment is begun and then monitored monthly

The nurse is reviewing the care of a patient with a pulmonary fungal infection. Which of these statements is true? 1 The patient will be placed on droplet isolation. 2 Opportunistic fungal infections occur in otherwise healthy people. 3 Pulmonary fungal infections are transmitted from person to person. 4 Amphotericin B is the standard therapy for treating serious systemic fungal infections.

4 RATIONALE: Amphotericin B remains the standard therapy for treating serious systemic fungal infections. These infections are not transmitted from person to person, and the patient does not have to be placed in isolation. Opportunistic fungal infections occur in immunocompromised patients (e.g., those being treated with corticosteroids, chemotherapy, and immunosuppressive drugs) and in patients with human immunodeficiency virus (HIV) and cystic fibrosis. Fungal infections are acquired by inhalation of spores.

A patient presents with a lung abscess. What treatment option would be the most appropriate? 1. Postural drainage 2. Chest physiotherapy 3. Reduction of fluid intake 4. Antibiotic treatment

4 RATIONALE: As there are mixed bacteria in a lung abscess, starting a broad spectrum antibiotic is the appropriate treatment option. Postural drainage and chest physiotherapy are not recommended, as they may cause spillage of infection to other bronchi and spread the infection. Reducing fluid intake is not advisable; instead, adequate fluid intake is recommended.

The emergency department nurse is assessing a patient who has been having trouble breathing for the last few months. His wife tells the nurse that he worked in a coal mine for 25 years but stopped working there last year. The nurse will look for manifestations of which problem? 1. Pulmonary edema 2. Pulmonary fibrosis 3. Respiratory alkalosis 4. Hypersensitivity pneumonitis

4 RATIONALE: Black lung is caused by inhalation of large amounts of coal dust, an occupational hazard for underground coal miners. The inhaled substance is ingested by macrophages, which releases substances that cause cell injury and death. Fibrosis occurs as a result of tissue repair. Repeated exposure eventually results in diffuse pulmonary fibrosis (excess connective tissue). Pulmonary edema and respiratory alkalosis are not caused by inhaled coal dust.

A patient presenting with pneumonia scores 5 on the CURB-65 scale. What action should the nurse take? 1. Advise no treatment. 2. Advise treating at home. 3. Consider hospital admission. 4. Consider admission to an intensive care unit.

4 RATIONALE: CURB-65 is used in addition to clinical judgment in determining the severity of pneumonia and the need for advanced medical care. A patient scoring 5 on the CURB-65 scale means the condition is severe and needs advanced medical care. Hence, the nurse should consider admission to an intensive care unit. If the patient has symptoms of pneumonia, advising no treatment is not an option. Treating at home is advised when the score on CURB-65 scale is 0. Hospital admission is considered when the score on the CURB-65 scale is 1-2.

The nurse is planning to deflate the tracheostomy cuff of a patient. Which task is least helpful in preventing secretions from being aspirated during deflation? 1. Suction the tracheostomy tube 2. Have the patient cough up secretions 3. Suction the mouth 4. Deflate the cuff during inspiration

4 RATIONALE: Deflating the cuff during inspiration would be least helpful in preventing aspiration because the tracheostomy cuff should be deflated during the patient's expiration and exhaled gas helps propel secretions into the mouth. Suctioning the tracheostomy tube and then the mouth are important steps in preventing secretions from being aspirated during cuff deflation. Having the patient cough and therefore clear the lower airway before cuff deflation minimizes risk of aspiration. Suctioning the tracheostomy tube and then the mouth are important steps in preventing secretions from being aspirated during cuff deflation.

The nurse understands that one way to assess the degree of impairment in gas exchange is to measure the arterial oxygen/fraction of inspired oxygen (PaO2/FIO2, or P/F) ratio. What is the P/F ratio in acute lung injury (ALI)? 1. Greater than 400 2. Greater than 300 3. Less than 200 4. Between 200 and 300

4 RATIONALE: In ALI, the P/F ratio is between 200 and 300. This indicates compromised gas exchange through the alveoli. Under normal circumstances, when PaO2 is 85 to 100 mm Hg and FIO2 is 0.21, the P/F ratio would be greater than 400. The term acute respiratory distress syndrome (ARDS) is used when the P/F ratio is less than 200 (e.g., 80/0.8 = 100) and indicates refractory hypoxemia.

The nurse is assessing a patient with a pneumothorax. The nurse expects to note which type of fremitus? 1. Normal fremitus 2. Decreased fremitus 3. Increased fremitus 4. Absent fremitus

4 RATIONALE: In a patient with pneumothorax, the nurse would find absent fremitus. Decreased fremitus may be found in pleural effusion when the hand is farther from the lung, and in barrel chest where the lung is hyperinflated. Increased fremitus is found in pneumonia, in lung tumors, with thick bronchial secretions, and above a pleural effusion. As the patient's voice moves through a dense tissue or fluid-filled lungs, the vibration is increased.

A patient with human immunodeficiency virus (HIV) infection has been admitted with pneumonia. The nurse anticipates that this patient will receive treatment for which type of pneumonia? 1 Aspiration pneumonia 2 Hospital-associated pneumonia 3 Community-acquired pneumonia 4 Opportunistic pneumonia caused by Pneumocystis jiroveci

4 RATIONALE: Individuals at risk for opportunistic pneumonia include those with altered immune responses, such as HIV infection. In addition to the risk of bacterial and viral pneumonia, the immunocompromised person may develop an infection from microorganisms that do not normally cause disease, such as P. jiroveci (formerly carinii ). The patient likely does not have aspiration pneumonia, hospital-associated pneumonia, or community-acquired pneumonia.

The nurse is teaching a patient stoma care after a tracheostomy. Discharge instructions should include: 1. No diving into water but swimming is okay. 2. Washing the area around the stoma at least three times a day. 3. No covering of the stoma while coughing so secretions can be excreted. 4. Wearing a Medic Alert bracelet or other form of emergency identification

4 RATIONALE: It is important to wear a Medic Alert bracelet or other form of identification so that emergency personnel are able to identify that the patient breathes from the neck. Swimming and diving are contraindicated. At least once a day, the area around the stoma needs to be cleansed with a moist cloth. The stoma needs to be covered when coughing to prevent the spread of mucous and secretions to others.

The nurse is caring for a patient with a diagnosis of idiopathic pulmonary arterial hypertension (IAPH). The nurse knows that: 1. IAPH occurs more often in males than females. 2. The mean diagnosis time between onset of symptoms and diagnosis is 5 to 10 years. 3. IAPH increases the work of the left ventricle and causes left-ventricular hypertrophy. 4. Functional classification of IAPH is measured by using the New York Heart Association Scale

4 RATIONALE: Patients are classified using the New York Heart Association functional classification. IAPH occurs more often in females than in males. The time between onset of symptoms and diagnosis is about two years and usually by the time the patient becomes symptomatic, the disease is in the advanced stages. IAPH increases the work of the right ventricle and causes right-ventricular hypertrophy (cor pulmonale).

The nurse is caring for a patient with a shunt due to acute respiratory distress syndrome (ARDS). Which nursing intervention is associated with better symptomatic relief for this patient? 1 Mechanical ventilation only 2 Bronchodilators along with corticosteroids 3 High fraction of inspired oxygen (FIO2) only 4 Mechanical ventilation and high FIO2

4 RATIONALE: Patients with ARDS having a shunt disorder are usually more hypoxemic than patients with ventilation-perfusion (V/Q) mismatch. They often require mechanical ventilation and a high FIO2 in combination to improve gas exchange. Bronchodilators and corticosteroids are not helpful for immediate relief because the patient does not have bronchospasm and inflammation.

A patient with emphysema is receiving oxygen at 1 L/min by way of nasal cannula. The nurse understands that this prescription is appropriate because: 1 The patient does not require more than 1 L of oxygen 2 High concentrations of oxygen may rupture the alveoli 3 Oxygen is the natural stimulus for breathing and not required 4 High concentrations of oxygen eliminate the respiratory drive

4 RATIONALE: Patients with emphysema become accustomed to a high level of carbon dioxide and low level of oxygen. This situation reverses the natural breathing stimulus. A low oxygen level then becomes the stimulus for breathing, and too much oxygen will eliminate the stimulus to breathe. There is not enough information to determine that the patient does not need more than 1 L of oxygen. A high concentrations of oxygen does not rupture alveoli. In healthy individuals, increased carbon dioxide, not oxygen, is the stimulus for breathing.

Which nursing intervention is used to optimize oxygenation and ventilation in a patient with acute respiratory failure? 1 Suction the patient every hour. 2 Avoid hyperventilating the patient. 3 Position the patient with the good lung up. 4 Provide adequate rest and recovery time between procedures.

4 RATIONALE: Providing adequate rest and recovery time between various procedures prevents desaturation and optimizes oxygenation. Suctioning is indicated when the patient's condition warrants it, not when it is scheduled regularly. Hyperventilation may be used before treatments, repositioning, interventions, and so forth to allow the patient to have a "reserve." Positioning for optimal oxygenation and ventilation is good lung down, not up.

The nurse teaches pursed-lip breathing to a patient with emphysema. The nurse explains that the primary reason for this technique is: 1. Conserving energy 2. Relieving chest pain 3. Promoting oxygen saturation 4. Promoting carbon dioxide elimination

4 RATIONALE: Pursed-lip breathing promotes elimination of carbon dioxide by increasing positive pressure within the alveoli and making it easier to expel air from the lungs. This procedure also helps the patient slow the breathing and increase the depth of respirations. The purpose of instructing the patient in pursed-lip breathing is not to conserve energy, reduce chest pain, or promote oxygen saturation, although when performed correctly it may result in improved oxygenation.

The nurse is caring for a patient who has had a left pneumonectomy. An appropriate nursing intervention for a patient postpneumonectomy is 1. Monitoring chest tube drainage and functioning 2. Positioning the patient on the nonoperative side 3. Auscultating frequently for lung sounds on the operative side 4. Encouraging range-of-motion exercises on the affected upper extremity

4 RATIONALE: Range of motion exercises performed on the affected upper extremity will prevent edema and encourage circulation to the lung space to promote healing. A patient who has had a pneumonectomy may have a clamped chest tube postoperatively, so there will not be any drainage. Fluid will gradually fill space where the lung has been removed. Position patient on operative side to facilitate expansion of remaining lung. There will not be lung sounds on the operative side as the entire lung has been removed.

A patient who has tuberculosis (TB) is being treated with combination drug therapy. The nurse explains that combination drug therapy is essential because: 1. It minimizes the required dosage of each of the medications 2. It helps reduce the unpleasant side effects of the medications 3. It shortens amount of time that the treatment regimen will be needed 4. It discourages the development of resistant strains of the TB organism

4 RATIONALE: Recommendations for the initial treatment of tuberculosis (TB) includes a four-drug regimen until drug susceptibility tests are available. After susceptibility is established, the regimen can be altered, but patients should still receive at least two drugs to prevent emergence of drug-resistance organisms. Dosage, side effects, and duration of the regimen are not reasons for combination drug therapy in a patient with TB.

A patient who has just undergone a laryngectomy wants to lie flat in bed. Which statement would be an incorrect explanation for placing the patient in semi-Fowler position? 1. Semi-Fowler position will limit tension on the suture lines 2. Semi-Fowler position will decrease edema 3. Semi-Fowler position will help maintain airway patency 4. Semi-Fowler position will enhance the ability to communicate

4 RATIONALE: Semi-Fowler position will not enhance one's ability to communicate. After a laryngectomy, the patient will need teaching about methods of communicating other than speaking, either temporarily or permanently. Semi-Fowler position will limit tension on the suture lines, decrease edema, and help maintain airway patency are all correct. Maintenance of a patent airway is essential after laryngectomy. This is achieved primarily by keeping the patient in semi-Fowler position postoperatively. Keeping the patient in semi-Fowler position decreases edema and limits tension on the suture lines. This is one of many correct explanations for keeping the patient in semi-Fowler position postlaryngectomy.

The patient with asthma asks the nurse, "How will I know when my inhaler is empty?" What is the best response by the nurse? 1. "There will be no sound when shaking the canister." 2. "The canister will float in water." 3. "Your wheezing will not improve despite inhaler use." 4. "You need to keep track of how many puffs you have used and how many puffs are available."

4 RATIONALE: Show the patient how to determine how many puffs are available in the canister. Then teach the patient to document each time a puff is used. It is important to teach the patient this simple way to check the inhaler. The canister may or may not produce a sound when shaking. Floating the canister in water used to be recommended, but is no longer recommended because water can enter the chamber. It is not reasonable to wait for the patient's wheezing to worsen before getting a new inhaler.

The nurse is caring for a patient with a non-massive pulmonary embolism (PE). What is the best standard for treatment? 1. Tissue plasminogen activator (tPA) 2. Alteplase (Activase) 3. Warfarin (Coumadin) 4. Enoxaparin (Lovenox)

4 RATIONALE: Subcutaneous administration of low-molecular-weight heparin has been found to be safer and more effective than use of unfractionated heparin. It is also the recommended choice of treatment for patients with non-massive PE. Criteria for fibrinolytic therapy in PE include hemodynamic instability and right ventricular dysfunction. Fibrinolytic therapy dissolves pulmonary embolisms and the source of the thrombus. Low molecular weight heparin (Lovenox) is becoming more common for non-massive pulmonary embolism. Warfarin should be initiated within the first 24 hours s of heparinization and is typically administered for three to six months. Warfarin therapy is the standard treatment for non-massive PE.

Which statement about a deviated septum is correct? 1. All septal deviations are symptomatic 2. A septoplasty will not correct a deviated septum 3. Epistaxis is a rare manifestation of a deviated septum 4. The aim of medical management is nasal allergy contro

4 RATIONALE: The aim of medical management is symptom control because medical management of a deviated septum is focused on symptom control of nasal inflammation and congestion. All septal deviations are symptomatic, a septoplasty will not correct a deviated septum, and an epistaxis is a rare manifestation of a deviated septum are incorrect because minor septal deviations are typically asymptomatic, septoplasty will properly align a deviated septum, and epistaxis is one of many common manifestations of septal deviation.

Infection can be a major hazard of O2 administration. Heated nebulizers present the highest risk. The most common organism found is: 1. Rickettsia prowazekii 2. Clostridium perfringens 3. Bordatella pertussis 4. Pseudomonas aeruginosa

4 RATIONALE: The constant use of humidity supports bacterial growth, with the most common organism being P. aeruginosa. Rickettsia prowazekii, Clostridium perfringens, and Bordatella pertussis are not the most common organisms found in this case.

The nurse evaluates that discharge teaching for a patient hospitalized with pneumonia has been effective when the patient makes which statement about measures to prevent a relapse? 1. "I will seek immediate medical treatment for any upper respiratory infections." 2. "I should continue to do deep-breathing and coughing exercises for at least 12 weeks." 3. "I will increase my food intake to 2400 calories a day to keep my immune system well." 4. "I must have a follow-up chest x-ray in six to eight weeks to evaluate the pneumonia's resolution."

4 RATIONALE: The follow-up chest x-ray will be done in six to eight weeks to evaluate pneumonia resolution. A patient should seek medical treatment for upper respiratory infections that persist for more than seven days. It may be important for the patient to continue with coughing and deep breathing exercises for six to eight weeks, not 12 weeks, until all of the infection has cleared from the lungs. Increased fluid intake, not caloric intake, is required to liquefy secretions.

The nurse concludes that interventions carried out to promote airway clearance in a patient admitted with asthma are successful on the basis of which finding? 1. Oxygen saturation 96% 2. Use of accessory muscles 3. Absence of wheezing 4. Clearance of mucous from the bronchi

4 RATIONALE: The issue is airway clearance, which is evaluated most directly as successful if the patient can engage in effective and productive coughing. Oxygen saturation would indicate gas exchange, not airway clearance. Use of accessory muscles indicates respiratory distress. The absence of wheezing does not always coincide with improved airway clearance, and may represent worsening bronchospasm

The patient seeks relief from the symptoms of an upper respiratory infection (URI) that has lasted for five days. Which patient assessment should the nurse use to help determine if the URI has developed into acute sinusitis? 1 Coughing 2 Fever, chills 3 Dust allergy 4 Maxillary pain

4 RATIONALE: The nurse should assess the patient for sinus pain or pressure as a clinical indicator of acute sinusitis. Coughing and fever are nonspecific clinical indicators of a URI. A history of an allergy that is likely to affect the upper respiratory tract is supportive of the sinusitis diagnosis, but is not specific for sinusitis.

The patient has decided to use the voice rehabilitation that offers the best speech quality even though it must be cleaned regularly. The nurse knows that this is what kind of voice rehabilitation? 1. Electromyograph 2. Intraoral electrolarynx 3. Neck type electrolarynx 4. Transesophageal puncture

4 RATIONALE: The transesophageal puncture provides a fistula between the esophagus and trachea with a one-way valved prosthesis to prevent aspiration from the esophagus to the trachea. Air moves from the lungs, vibrates against the esophagus, and words are formed with the tongue and lips as the air moves out the mouth. The electromyography and both electrolarynx methods produce low-pitched mechanical sounds.

The nurse is assisting the health-care provider (HCP) perform a diagnostic thoracentesis on a patient. The nurse positions the patient in what position? 1 Lying flat in the fetal position on the unaffected side 2 Sitting in bed with knees slightly flexed and feet flexed 3 Lying flat on the unaffected side with knees slightly flexed 4 Sitting upright with elbows on an over bed table and feet supported

4 RATIONALE: To appropriately locate the pleural space, the patient needs to be positioned sitting upright with elbows on an over bed table, with feet supported. Lying flat would not adequately expand the thorax and permit the provider to position the thoracentesis needle in the correct place. Sitting in bed would also not allow the HCP to visualize the pleural space for needle insertion.

A patient with a new laryngectomy is considering different methods of voice restoration. The nurse knows that which of these offers the best speech quality and patient satisfaction? 1. Esophageal speech 2. Intraoral electrolarynx 3. Neck type electrolarynx 4. Transesophageal puncture

4 RATIONALE: Transesophageal puncture, the most common voice rehabilitation method, offers the best speech quality with the highest patient satisfaction. Esophageal speech is used, but very few develop fluent speech. Neck type electrolarynx and intraoral electrolarynx are easy to use and common, but have a mechanical-sounding speech.

The nurse is teaching a patient how to self-administer ipratropium (Atrovent) via a metered dose inhaler (MDI). Which instruction given by the nurse is most appropriate to help the patient learn the proper inhalation technique? 1 "Avoid shaking the inhaler before use." 2 "Breathe out slowly before positioning the inhaler." 3 "Using a spacer should be avoided for this type of medication." 4 "After taking a puff, hold the breath for 30 seconds before exhaling."

4 RATIONALE: It is important to breathe out slowly before positioning the inhaler. This allows the patient to take a deeper breath while inhaling the medication, thus enhancing the effectiveness of the dose. The inhaler should be shaken well. A spacer may be used. Holding the breath after the inhalation of medication helps keep the medication in the lungs, but 30 seconds will not be possible for a patient with COPD. Text Reference - p. 573

Which statement made by the patient with chronic obstructive pulmonary disease (COPD) indicates a need for further teaching regarding the use of an ipratropium inhaler? 1 "I can rinse my mouth following the two puffs to get rid of the bad taste." 2 "I should wait at least one to two minutes between each puff of the inhaler." 3 "Because this medication is not fast acting, I cannot use it in an emergency if my breathing gets worse." 4 "If my breathing gets worse, I should keep taking extra puffs of the inhaler until I can breathe more easily."

4 The patient should not just keep taking extra puffs of the inhaler to make breathing easier. Excessive treatment could trigger paradoxical bronchospasm, which would worsen the patient's respiratory status. Rinsing the mouth after the puffs will eliminate a bad taste. Waiting one to two minutes between each puff will facilitate the effectiveness of the administration. Ipratropium is not used in an emergency for COPD. Text Reference - p. 589

An elderly patient is in end-stage chronic lung disease due to chronic obstructive pulmonary disease (COPD). What nursing interventions are appropriate for this patient? Select all that apply. 1 Encourage patient to perform chest exercises. 2 Elevate the foot of the bed. 3 Use benzodiazepines to reduce anxiety. 4 Immediately report any change in mental status. 5 Monitor specific and nonspecific signs of respiratory failure.

4,5 While caring for patients with end-stage chronic lung disease, the nurse must immediately report any change in mental status such as agitation or confusion. This may indicate the onset of rapid deterioration in clinical status and the need for mechanical ventilation. It is especially important to monitor specific and nonspecific signs of respiratory failure in patients with COPD because a small change can cause significant decomposition. Chest exercises are needed only for rehabilitation once the crisis has passed. Elevation of the foot of the bed increases respiratory distress; instead, head end elevation is useful. Although benzodiazepines reduce anxiety, they suppress the respiratory center.

Routine tracheostomy care is on the treatment plan for a patient that has a fenestrated tracheostomy tube. What should be the order of actions performed by the nurse for proper tracheostomy care? 1.Clean the stoma using 4 x 4 gauze pad and the tracheostomy faceplate using cotton swabs. 2.Wash hands and put on gloves and goggles. 3.Remove fenestrated inner cannula and rinse in a sterile solution, and replace it. 4.Change tracheostomy ties using a two-person change technique. 5.Maintain the position of the retention sutures near the stoma. 6.Assemble sterile equipment

6,2,3,1,5,4 RATIONALE: During routine tracheostomy care, the required sterile equipment should be assembled near the patient's bed. Hands should be washed before the procedure. Clean gloves and goggles are to be worn. At first, the inner cannula is unlocked and cleaned in sterile water or saline solution. The inner cannula should be replaced after cleaning. The stoma is then cleaned with sterile gauze to remove dried secretions. Retention sutures, if present, should be properly positioned and secured. Subsequently, tracheostomy ties are changed.

The nurse would monitor which comorbidity in the patient treated for an asthma exacerbation with methylprednisolone (SoluMedrol)? 1. Diabetes mellitus 2. Hyperlipidemia 3. Hypothyroidism 4. Raynaud's phenomenon

1 RATIONALE: Hyperglycemia or increased blood glucose level is an adverse effect of methylprednisolone, so the patient with diabetes mellitus should be monitored for elevations in blood sugar. Methylprednisolone will not affect elevated cholesterol, hypothyroidism, or Raynaud's phenomenon.

To ease pleuritic pain caused by pneumonia, what nursing interventions should be performed? 1. Instructing the patient to splint the chest when coughing 2. Offering the patient an incentive spirometer every 4 hours 3. Instructing the patient in how to perform abdominal breathing 4. Encouraging the patient use shallow breathing during episodes of pain

1 RATIONALE: Pleuritic chest pain is triggered by chest movement and is particularly severe during coughing and deep breathing. Splinting the chest wall will reduce movement and thus ease discomfort during coughing. Use of an incentive spirometer and practicing abdominal breathing may help increase respiratory efficiency, remove secretions, and increase oxygenation, but it will not ease pleuritic pain. The patient should not be encouraged to breathe shallowly, because this will increase the risk for atelectasis and decrease oxygenation.

The nurse expects to find which pathological change in a patient with pulmonary arterial hypertension? 1 Increased pulmonary vascular resistance 2 Decreased pulmonary vascular resistance 3 Respiratory acidosis with failure to compensate 4 Increased crackles upon auscultation of the lower posterior and lateral lungs

1 RATIONALE: The most common cause of pulmonary arterial hypertension (PAH) is increased pulmonary vascular resistance due to primary disease or as a secondary complication of respiratory, cardiac, autoimmune, hepatic, or connective tissue disorders. Increased pulmonary vascular resistance is caused by vasoconstriction, remodeling (vessel wall thickening), and thrombosis. PAH causes right ventricular hypertrophy and eventually right-sided heart failure. PAH does not cause decreased pulmonary vascular resistance or respiratory acidosis. Increased crackles in any area of the lungs would be present in left-sided heart failure, not right-sided heart failure.

A 46-year-old patient who has undergone total left-knee arthroplasty complains of shortness of breath and slight chest pain. Temperature is 98°F, blood pressure 140/86 mmHg, respirations 30, and oxygen saturation 92% on room air. The nurse suspects that the patient is experiencing which condition? 1 Pulmonary embolus 2 Unstable angina 3 Chronic obstructive pulmonary disease (COPD) exacerbation 4 Pneumonia

1 RATIONALE: The patient presents the classic symptoms of pulmonary embolus: acute onset of symptoms, tachypnea, shortness of breath, and chest pain. Unstable angina would present with chest pain occurring at rest; COPD exacerbation would present with wheezing, cough, and shortness of breath. Pneumonia would be evident if the patient had a fever, elevated white blood cell count, and a productive cough with yellow, green, or rust-colored sputum.

The nurse is caring for a patient with obstructive pulmonary disease who had tachycardia, tachypnea, and restlessness. The patient now becomes very lethargic and has a normal respiratory rate. How does the nurse interpret this? 1.Worsening of the condition 2. Stabilization of the condition 3. Improvement in the condition 4. A more relaxed and resting patient

1 RATIONALE: The patient's condition has deteriorated as evidenced by lethargy and decreased respiratory rate. The elevated carbon dioxide levels have affected the central nervous system, causing lethargy, which may progress to coma. The patient has become exhausted and is unable to maintain the compensatory mechanisms needed to maintain acid-base balance. This patient is not improving nor is more relaxed and resting. This patient's condition is not stabilizing.

The nurse is preparing a patient for video-assisted thoracotomy surgery (VATS). The nurse knows that this type of procedure: 1 Is a minimally invasive surgical approach 2 Provides a real-time one-dimensional video 3 Is contraindicated in patients with chest trauma 4 Can only be used for patients in previously good health

1 RATIONALE: VATS is a minimally invasive surgical approach to the chest cavity to diagnose and treat diseases of the pleura, pulmonary masses and nodules, mediastinal masses, and interstitial lung disease. VATS involves a real-time two-dimensional video. VATS is a good option for debilitated patients and those with limited respiratory reserve. It is being used also for patients with chest trauma.

The nurse is reviewing gender, cultural differences, and ethnic differences of patients with obstructive lung disease. The nurse knows that: 1. Whites have the highest incidence of cystic fibrosis. 2. Before puberty, more girls are affected with asthma than boys. 3. Male African Americans have the highest mortality rates from asthma than all ethnic/gender groups. 4. African Americans and Hispanics have a higher incidence of chronic obstructive pulmonary disease (COPD).

1 RATIONALE: Whites have the highest incidence of cystic fibrosis. Before puberty, boys are affected more with asthma than girls. Female African Americans have the highest mortality rates from asthma than all ethnic/gender groups. Whites have the highest incidence of (COPD), even though higher rates of smoking occur in other ethnic groups.

What is the most common sign during an initial assessment that alerts the nurse that the patient has chronic obstructive pulmonary disease? 1. Barrel chest 2. Sunken Chest 3. Hyperventilation 4. Circumoral cyanosis

1 RATIONALE: The patient with chronic obstructive pulmonary disease (COPD) develops a barrel chest over time because trapped air enlarges the lungs and thoracic cavity, thereby reducing chest flexibility. Sunken chest, also known as funnel chest or pectus excavatum, is not related to COPD. Hyperventilation is not characteristically seen with COPD. Instead, the patient usually displays persistent dyspnea on exertion, with or without a chronic cough. Circumoral cyanosis is a bluish discoloration of the skin surrounding the mouth. It is usually an indication of a severely diminished level of oxygen and respiratory distress. Circumoral cyanosis can result from a variety of respiratory diseases and may be a late sign of the COPD disease process.

A nurse is caring for a pediatric patient who has experienced a cough, clear and watery sputum, headache, and muscle aches for the past two weeks. The nurse auscultates wheezes during expiration. There is no other abnormality found. The patient's parent asks why an antibiotic has not been prescribed. How should the nurse respond? 1. Explain that antibiotics are not required for the patient. 2. Advise the parent to see another health care provider for a second opinion. 3. Explain that the child needs anticancer treatment, and antibiotics will not help. 4. Explain that antibiotics will be prescribed if the cough persists for two more days.

1 RATIONALE: The symptoms and signs indicate that the patient may have acute bronchitis, which is a viral disorder. Therefore the nurse should explain to the father that antibiotics will not help in viral infections. If they are prescribed, antibiotics may cause side effects and may also lead to antibiotic-resistance. It is incorrect to advise the father to see another healthcare provider who will do the same. It is also inappropriate to tell him that his child needs anticancer treatment. Acute bronchitis is a self-limiting disorder, and cough may last up to three weeks. Informing the father that antibiotics will be prescribed if the cough persists for two more days is not correct.

Which lung cancer diagnosis is associated with the most rapid growth rate? 1 Small cell carcinoma 2 Squamous cell carcinoma 3 Large cell carcinoma 4 Adenocarcinoma

1 RATIONALE; Small cell carcinoma is the most malignant form of lung cancer, with a very rapid growth rate. Squamous cell carcinoma has a slow growth rate, owing to its tendency to not metastasize. Large cell carcinoma is highly metastatic via the lymphatics and blood, but its growth rate is not as rapid as small cell. Adenocarcinoma is the most common type of lung cancer, a non-small cell lung cancer.

The nurse is observing a new graduate nurse during suctioning of a tracheostomy patient. Which action by the graduate nurse would require intervention? 1 Applying suction while inserting the catheter. 2 Limiting the suction time to 10 seconds or less. 3 Assessing the patient's SpO2 and heart rate and rhythm before the procedure. 4 Providing preoxygenation for a minimum of 30 seconds before the procedure.

1 RATIONALE: Suction is NOT applied while inserting the catheter; intermittent suction is applied as the catheter is being withdrawn. Preoxygenation is also required before suctioning, and suction time should be limited to 10 seconds or less. A patient is at risk for hypoxemia after suctioning. Therefore it is imperative to monitor the patient's oxygen status before, during, and after suctioning. The catheter should be rotated during withdrawal.

A nurse is monitoring the SvO2 of a patient. The SvO2 value is 50%. What does this value indicate? 1 Less oxygen is being delivered to the tissues. 2 Less oxygen is being consumed by the tissues. 3 The cardiac output is normal. 4 The patient is hypoventilating

1 RATIONALE: SvO2 is the measure of venous oxygen saturation. It can be measured through a pulmonary artery catheter. The normal values range between 60% to 80%. A decrease in SvO2 indicates that less oxygen is being delivered to the tissues, causing tissue hypoxia. A decrease in SvO2 also indicates that more oxygen is being consumed by the tissues, causing an imbalance of demand and consumption. It is an early indicator of a change in the cardiac output. It does not indicate hypoventilation; a measurement of CO2 is more appropriate for measuring hypoventilation.

A patient reports shortness of breath one day after a cholecystectomy. On examination there is dullness on percussion on the right side of the chest, and breath sounds are also decreased in this region. The nurse recognizes that the most probable reason for the assessment findings is what? 1. Atelectasis 2. Pneumonia 3. Pneumothorax 4. Tension pneumothorax

1 RATIONALE: Atelectasis is the most common complication seen after thoracic or abdominal surgery. In this condition the alveoli are collapsed, and there is no air in them. On examination the affected area is dull when percussion is done, and the breath sounds in the affected area are decreased. Therefore this patient most probably has atelectasis. Pneumonia can have similar findings, but it is highly unlikely to occur one day after surgery. In both pneumothorax and tension pneumothorax, the affected area is hyperresonant.

During admission of a patient diagnosed with non-small-cell lung carcinoma, the nurse questions the patient related to a history of which risk factors for this type of cancer? Select all that apply 1. Asbestos exposure 2. Exposure to uranium 3. Chronic interstitial fibrosis 4. History of cigarette smoking 5. Geographic area in which the patient was born 6. New Choice

1, 2, 4 RATIONALE: Non-small-cell carcinoma is associated with cigarette smoking and exposure to environmental carcinogens, including asbestos and uranium. Chronic interstitial fibrosis is associated with the development of adenocarcinoma of the lung. Exposure to cancer-causing substances in the geographic area where the patient has lived for some time may be a risk, but not necessarily where the patient was born.

What patient positioning strategies should the nurse use while caring for a patient with acute respiratory distress syndrome (ARDS)? Select all that apply. 1, Prone position 2, Supine position 3, Continuous lateral rotation therapy 4. Kinetic therapy 5. Lateral positioning

1, 3, 4 RATIONALE: Some patients with ARDS have a marked improvement in arterial oxygen (PaO2) when turned from the supine to the prone position with no change in fraction of inspired oxygen (FIO2). Continuous lateral rotation therapy and kinetic therapy are other strategies that can be used for patients with ARDS. Continuous lateral rotation therapy involves continuous, slow, side-to-side turning of the patient by rotating the actual bed frame less than 40 degrees. Kinetic therapy involves rotating the patient side to side 40 degrees or more. In a supine position, the heart and the mediastinal mass may put pressure on the lungs, predisposing the patient to atelectasis. Therefore, a supine position is not advisable. Lateral positioning is also not suitable, as it may cause pooling of secretions.

A nurse is providing care to a patient with hypoxemia secondary to an intrapulmonary shunt. Which nursing interventions provide better improvement in the patient's condition? Select all that apply 1. Positive pressure ventilation (PPV) via tight-fitting mask 2. Benzodiazepines to reduce intrapulmonary shunt 3.24% to 32% oxygen by face mask or Venturi mask 4. PPV via an endotracheal tube 5. Oxygen administration at 1 to 3 L/minute by nasal cannula

1, 4 RATIONALE: Hypoxemia secondary to an intrapulmonary shunt usually requires PPV. PPV provides oxygen therapy and humidification, decreases the work of breathing, and reduces respiratory muscle fatigue. In addition, PPV assists in opening collapsed airways and decreasing shunt. PPV is provided via an endotracheal tube (most frequently) or noninvasively by means of a tight-fitting mask. Benzodiazepines are an antianxiety drug and do not reduce intrapulmonary shunt. 24% to 32% oxygen by face mask or Venturi mask and oxygen administration at 1 to 3 L/minute by nasal cannula does not improve shunt. TEST-TAKING TIP: Come to your test prep with a positive attitude about yourself, your nursing knowledge, and your test-taking abilities. A positive attitude is achieved through self-confidence gained by effective study. This means (a) answering questions (assessment), (b) organizing study time (planning), (c) reading and further study (implementation), and (d) answering questions (evaluation).

The nurse is caring for a patient on positive pressure ventilation. What nursing interventions help to maintain cardiac output in this patient? Select all that apply. 1 Administer crystalloid fluids. 2 Administer colloid solutions. 3 Administer packed red blood cells. 4 Administer metoprolol (Lopressor). 5 Increase peak end expiratory pressure (PEEP)

1,2 RATIONALE: If cardiac output falls, it may be necessary to administer crystalloid fluids or colloid solutions to expand the volume and maintain hemodynamic stability. Packed red blood cells are used to increase tissue perfusion. Metoprolol (Lopressor) reduces cardiac contractility. PEEP has to be lowered to maintain cardiac output.

A nurse is educating a patient regarding hypercapnic respiratory failure. What information should the nurse include in her teaching? Select all that apply. 1. It is a ventilatory failure with insufficient carbon dioxide (CO2) removal. 2. There is evidence of the body's inability to compensate for acidemia. 3. Arterial CO2 (PaCO2) is greater than 45 mm Hg in combination with an arterial pH of less than 7.35. 4. The pH is at a level where a further increase may lead to severe acid-base imbalance. 5. There is inadequate oxygen transfer between the alveoli and the pulmonary capillaries.

1,2,3 RATIONALE: Hypercapnic respiratory failure is a ventilatory failure due to inadequate removal of CO2 from the body. The body may be unable to compensate for the acidemia caused by increased CO2 levels. As a result, the PaCO2 would be greater than 45 mm Hg in combination with acidemia (arterial pH less than 7.35). If the pH decreases further, it may lead to severe acid-base imbalance. Inadequate oxygen transfer between the alveoli and the pulmonary capillaries is a pathophysiological phenomenon during hypoxemic respiratory failure.

A patient with chronic obstructive pulmonary disease (COPD) is advised to use oxygen therapy at home. A nurse provides discharge instructions about how to prevent respiratory infection. What should be included in the teaching? Select all that apply. 1. Change the nasal cannula every 2 weeks. 2. Wash the nasal cannula twice a week. 3. Use mouthwash several times a day. 4. Retain secretions that are coughed out. 5. Clean the oxygen concentrator cabinet every week

1,2,3 RATIONALE: The strategies to reduce infection while using oxygen therapy at home include changing the cannula every 2 weeks, washing the cannula twice a week with liquid soap, and using a mouthwash several times a day. The nasal cannula may become contaminated with repeated use and should be changed every week. It should also be cleaned twice a week to remove the particulate material and moisture. Frequent use of mouthwash helps to keep the oral cavity clean and prevent infection. Removing the secretions that are coughed out reduces the risk of infection. The oxygen concentrator cabinet should be cleaned every day, not weekly

A patient is being admitted with a diagnosis of pertussis. The nurse knows which of the following? Select all that apply. 1. The cough may last from 6 to 8 weeks. 2. This is a highly contagious respiratory tract infection. 3. Treatment usually includes antibiotics with macrolides. 4. Cough suppressants and antihistamines should not be used. 5. Corticosteroids and bronchodilators are very useful in reducing symptoms. 6. Lifetime immunity results from one vaccination of diphtheria, pertussis, or tetanus (DPT).

1,2,3,4 RATIONALE: Pertussis is very contagious, and strict respiratory (droplet) precautions need to be instituted. Macrolide antibiotics, like erythromycin and azithromycin, usually are given to minimize symptoms and prevent the spread of the infection. The cough can last 6 to 8 weeks, but cough suppressants and antihistamines should not used because they are ineffective and can aggravate coughing episodes. Pertussis is caused by a gram-negative bacillus called Bordella pertussis. Corticosteroids and bronchodilators are not useful in reducing symptoms. Lifetime immunity does not result from one vaccination of DPT.

A patient is diagnosed with allergic rhinitis. Which instructions should the nurse include when teaching the patient about ways to avoid allergens? Select all that apply. 1. Wash your bedding in hot water weekly. 2. Remove pets from the interior of your home. 3. Ventilate closed rooms and open doors. 4.Wear a mask while vacuuming at home 5. Maintain high humidity levels at home

1,2,3,4 RATIONALE: Washing bedding in hot water weekly, removing pets from the interior of the home, ventilating closed rooms and opening doors, and wearing a mask while vacuuming are necessary to avoid allergens and will help to prevent allergic rhinitis. Maintaining high humidity levels at home will increase the chance of exposure to allergens and is not recommended.

The health care provider requests a computed tomography (CT) scan for a patient suspected of having a mediastinal mass. What should the nurse assess prior to the test? Select all that apply. 1 Blood urea nitrogen (BUN) 2 Serum creatinine 3 Blood sugar levels 4 Hypersensitivity to shellfish 5 Blood hemoglobin levels

1,2,4 RATIONALE The CT scan involves administering a contrast agent; therefore, the nurse should assess the renal function to determine the safety of contrast administration. The renal function can be assessed by checking the laboratory values of BUN and serum creatinine. The contrast agent is usually an iodine-based compound; therefore, the nurse should also check if the patient is hypersensitive to shellfish, as it contains iodine. Checking blood sugar levels and hemoglobin levels are important, but not necessary prior to a CT.

When teaching a patient with asthma about ways to reduce the severity of asthma and asthma attacks, which measures should be included? Select all that apply. 1 Avoid food irritants. 2 Avoid animals with fur. 3 Go out in the cold air for a walk. 4 Identify personal triggers. 5 Use nonsteroidal antiinflammatory drugs (NSAIDs)

1,2,4 RATIONALE: Patients with asthma should be taught to avoid food irritants and animals with fur. Identifying personal triggers can help to avoid them. Going out in the cold air and the use of NSAIDs are not recommended, because they can precipitate an asthma attack.

A 61-year-old patient with asthma is admitted to the hospital. The nurse understands that symptoms of asthma include which of the following? Select all that apply. 1. Wheezing 2. Chest tightness 3. Crackles 4. Cough 5. Pink frothy sputum

1,2,4 RATIONALE: Symptoms of asthma include cough, chest tightness, and wheezing. Crackles are heard when fluid has accumulated in the lungs, which is not consistent with asthma. Pink frothy sputum is seen with pulmonary edema

A nurse is caring for a patient with asthma. What instructions should the nurse provide to the patient to help prevent triggers of bronchoconstriction? Select all that apply. 1 Avoid wearing perfumes. 2 Avoid exposure to strong odors. 3 Do not drink cold water. 4 Avoid exposure to cold air. 5 Avoid people with certain infections.

1,2,4 RATIONALE: The different triggers for asthmatic attack include perfumes, strong odors, and cold air. They cause reflex bronchoconstriction, and the patient should avoid them to prevent asthma attacks. Cold water and bacteria do not trigger asthma.

The nurse is caring for the patient with chronic obstructive pulmonary disease (COPD). Which of the following are appropriate patient goals? Select all that apply. 1.Prevention of disease progression 2.Relief of symptoms 3.Healing of damaged lung tissue 4.Ability to perform activities of daily living (ADLs) 5. Improved quality of life

1,2,4,5 RATIONALE: The overall goals are that the patient with COPD will have (1) prevention of disease progression, (2) ability to perform ADLs and improved exercise tolerance, (3) relief from symptoms, (4) no complications related to COPD, (5) knowledge and ability to implement a long-term treatment regimen, and (6) overall improved quality of life. Lung tissue does not regenerate so "healing" is not a realistic goal. Patients need to know that symptoms can be managed, but COPD cannot be cured.

Complications of pneumonia occur more frequently in older patients. The nurse knows that potential complications include which of the following? Select all that apply. 1 Sepsis 2 Pleurisy 3 Bronchitis 4 Encephalitis 5 Pleural effusion 6 Congestive heart disease

1,2,5 RATIONALE: Pleurisy, or inflammation of the pleura, and sepsis with bacteria in the blood stream can result from pneumonia, as well as pleural effusion (fluid in the pleural space). Bronchitis and encephalitis are not complications. Congestive heart failure is not known directly as a complication of pneumonia

A senior nurse is teaching nursing students to auscultate for adventitious sounds. One of the students auscultates a pleural friction rub. What are the common conditions in which a pleural friction rub is present? Select all that apply. 1. Pleurisy 2. Pneumonia 3. Asthma 4. Bronchitis 5. Pulmonary infarct

1,2,5 RATIONALE: The most common conditions presenting with pleural rub are pleurisy, pneumonia, and pulmonary infarct. Pleural rub is caused by the rubbing together of the two layers of the lungs. Asthma and bronchitis present with wheezes and do not manifest as pleural rub.

A patient comes to the emergency department (ED) complaining of recurrent epistaxis. The nurse knows that recurrent episodes of epistaxis can be caused by which of the following? Select all that apply. 1.Nasal tumors 2.Facial trauma 3. High humidity 4.Fever of unknown origin 5. Foreign bodies inserted into nares 6. Overuse of nasal decongestant sprays

1,2,5,6 RATIONALE: Nasal tumors, some anatomic malformations of the nose, foreign bodies inserted into the nose, and facial trauma can result in epistaxis. Using nasal decongestant sprays too frequently can also cause nose bleeds. Fever of unknown origin does not cause nasal bleeding. Low humidity, not high humidity, also can increase one's risk of epistaxis.

The nurse is completing a respiratory assessment on a recently admitted patient who uses oxygen at home. What information does the nurse need to obtain and document on the patient? Select all that apply. 1. Liter flow 2. Home safety practices 3. Method of payment for oxygen services 4. What agency supplies the oxygen services 5. Method and effectiveness of administration 6. Fraction of inspired oxygen concentration (FiO2)

1,2,5,6 RATIONALE: How the patient administers the oxygen and its effectiveness, home use, liter flow and FiO2 value need to be determined and documented. It is also important to find out what home safety practices are followed. What oxygen service the patient uses and the patient's method of payment are not necessary to determine in the admission assessment.

The patient with a tracheostomy is receiving humidified air. Which rationale(s) for the patient's use of humidification are correct? Select all that apply. 1 Prevents formation of mucous plugs 2 Prevents lower airway heat loss 3 Warms secretions 4 Moisturizes secretions

1,3,4 RATIONALE: Preventing formation of mucous plugs, warming secretions, and moisturizing secretions are correct because humidification is essential to prevent retention of tenacious secretions and formation of mucous plugs. Humidified air compensates for the loss of the upper airway to warm and moisturize secretions. Preventing lower airway heat loss is incorrect because humidified air compensates for the loss of the upper airway to warm and moisturize secretions, not lower airway. Additionally, it actively warms, and therefore does not prevent heat loss.

A patient is scheduled for a biopsy through bronchoscopy. What nursing interventions are appropriate for this patient? Select all that apply. 1 Ensure that an informed consent has been signed. 2 Assess blood urea nitrogen (BUN) and serum creatinine levels. 3 Administer sedative, if prescribed. 4 Instruct patient to remove all metal from the body. 5 Instruct patient to be on NPO status for 6-12 hours before the test.

1,3,5 RATIONALE: The nurse should ensure the consent form has been signed and administer sedatives if prescribed. For a bronchoscopy, the patient is required to be on NPO for 6-12 hours before the procedure. Renal function is not related to bronchoscopy, so assessment of BUN and serum creatinine levels is not needed. Removing metals from the body may not be necessary, as the procedure does not involve x-rays.

What should be the overall nursing goal for a patient with acute respiratory failure? Select all that apply. 1 No dyspnea 2 Normal bowel sounds 3 Maintenance of the airway 4 Normal blood urea nitrogen and creatinine 5 Effective cough and ability to clear secretions

1,3,5 RATIONALE: The overall goals for the patient with acute respiratory failure are absence of dyspnea, maintenance of the airway, effective cough and ability to clear secretions, normal ABG values and normal breath sounds. Absence of dyspnea indicates normal breathing and adequate oxygenation. Maintenance of the airway is important for normal oxygenation and tissue perfusion. Effective coughing and the ability to clear secretions help to keep the airway patent. Normal bowel sounds are not a goal for respiratory failure, as they are related to the function of the gastrointestinal tract. Blood urea nitrogen and serum creatinine are markers of renal function, not respiratory function.

A patient with asthma has been advised to undergo a pulmonary function test. What actions should the nurse take to prepare the patient for the procedure? Select all that apply. 1, Explain the procedure to the patient. 2. Withhold liquids for 2 hours before the test. 3. Instruct patient to wear comfortable shoes. 4. Avoid scheduling the test immediately after a meal. 5. Avoid administration of inhaled bronchodilator 6 hours before the procedure.

1,4,5 RATIONALE: When preparing the patient for a pulmonary function test , the nurse should explain the procedure to the patient. The test should not be scheduled immediately after meals. The nurse should not administer an inhaled bronchodilator to the patient 6 hours before the procedure. It is not necessary to withhold liquids before the test, and the patient does not need to wear any particular type of shoes.

A patient with chronic obstructive pulmonary disease (COPD) reports waking up frequently from sleep. The patient smokes cigarettes before going back to sleep. What instructions should the nurse give to the patient to promote sleep? Select all that apply. 1 Do not smoke between periods of sleep. 2 Ask the health care provider about using diazepam (Valium) to induce sleep. 3 Ask the health care provider about using β2-agonists to relieve insomnia. 4 Do not use oxygen therapy at night. 5 Use saline nasal sprays before sleep if experiencing postnasal drip.

1,5 RATIONALE: Cigarette smoking aggravates sleep disturbance so the nurse should discourage this habit. The postnasal drip associated with COPD causes disturbed sleep and can be managed by using saline nasal sprays before going to sleep and upon awakening. Use of diazepam should be discouraged as it may suppress respiration. Use of β2-agonists will aggravate insomnia. If the patient is prescribed oxygen therapy, it should be continued as it helps to relieve insomnia.

A patient has a known allergy to dogs and is prescribed cromolyn spray (NasalCrom). What should the nurse instruct the patient regarding the use of the spray? 1 Use the spray daily. 2 Use the spray 15 minutes before contact with a dog. 3 Use the spray 15 minutes after contact with a dog. 4 Use the spray only when symptoms of rhinitis show up.

2 RATIONALE: Cromolyn is a mast cell stabilizer which inhibits the secretion of histamine and other inflammatory mediators. It is used to prevent symptoms of rhinitis and should be used 10 to 15 minutes before likely exposure to a known allergen. It is not recommended for daily use. Using the spray after allergen exposure would not be beneficial. Using the spray after the symptoms appear would also not help the patient.

The nurse is working in the hospital during the flu season and knows that: 1 The onset is insidious 2 Generalized myalgia occurs 3 Vomiting and diarrhea result 4 Nuchal rigidity starts before headache

2 RATIONALE: Generalized myalgia or body aches are common with flu symptoms. The onset of flu is abrupt and not insidious. Anorexia occurs, but not vomiting and diarrhea. Nuchal rigidity is impaired neck flexion resulting from muscle spasms of the neck and is related to meningeal irritation.

A patient needs to receive oxygen at a 28% concentration. The nurse will set up which oxygen delivery device? 1 Nasal cannula 2 Venturi mask 3 Simple face mask 4 Nonrebreather mask

2 RATIONALE: The Venturi mask is a high-flow device that delivers fixed concentrations of oxygen (e.g., 24%, 28%), independent of the patient's respiratory pattern). Nasal cannulas, simple face masks, and nonrebreather masks are examples of low-flow devices that deliver less precise concentrations of oxygen

A patient presents to the outpatient clinic with nasal trauma. On examination, the nurse finds localized pain, edema, epistaxis, and crepitus on palpation. The patient does not have nasal congestion or difficulty in breathing. Which conditions should the nurse suspect? 1.Acute sinusitis 2. Nasal fracture 3. Allergic rhinitis 4. Deviated nasal septum

2 RATIONALE: Trauma to the nose is the major cause of nasal fracture. It is manifested as localized pain, edema, epistaxis, and crepitus on palpation. Acute sinusitis presents with pain over the affected sinuses, purulent nasal discharge, and fever. Allergic rhinitis presents with sneezing, itchy eyes and nose, and watery nasal discharge. Deviated nasal septum presents with obstructed nasal breathing and nasal congestion. TEST-TAKING TIP Note the number of questions and the total time allotted for the test to calculate the times at which you should be halfway and three-quarters finished with the test. Look at the clock only every 10 minutes or so.

An asthmatic patient is in acute respiratory distress. The nurse auscultates the lungs and notes cessation of inspiratory wheezing. How does the nurse interpret this finding? 1 The patient has developed a pneumothorax 2 There is worsening airway inflammation and bronchoconstriction 3 Airflow has now improved through the bronchioles 4 A mucous plug has developed within a main stem bronchus

2 RATIONALE: When the patient in respiratory distress has inspiratory wheezing that then ceases, it is an indication of airway obstruction and requires emergency action to restore the airway.. A pneumothorax would be evidenced by absent breath sounds. Absence of wheezing does not correlate with improved airflow if the patient is also in current respiratory distress. A mucus plug would result in crackles in the lungs

The nurse determines that a patient is experiencing the most common adverse effect of albuterol (ProAir HFA) after noting which sign? 1. Headache 2.Tachycardia 3. Diarrhea 4. Oral candidiasis

2 RATIONALE: Tachycardia is a common adverse effect of the use of inhaled beta 2 adrenergic agonists because of its stimulant effect. Headache, diarrhea, and oral candidiasis are not associated adverse effects of albuterol. TEST-TAKING TIP: Choose the best answer for questions asking for a single answer. More than one answer may be correct, but one answer may contain more information or more important information than another answer.

A patient has an initial positive PPD (purified protein derivative) skin test result. A repeat PPD's result is also positive. No other signs or symptoms of tuberculosis or allergies are evident. Which medication(s) does the nurse anticipate will be prescribed? 1 Penicillin 2 Isoniazid (INH) 3 INH and an antibiotic 4 Theophylline (Theo-Dur)

2 RATIONALE: Isoniazid (INH) is the most commonly initially prescribed medication to treat patients exposed to tuberculosis (TB). Penicillin and theophylline would not be prescribed for treatment of TB exposure. INH plus an antibiotic would not likely be prescribed for this scenario.

The nurse caring for a patient with diabetes mellitus and chronic obstructive pulmonary disease (COPD) on oral prednisone (Deltasone) will monitor which parameter regularly? 1. Blood pressure 2. Blood sugar 3. Hemoglobin A1c 4. Bowel Sounds

2 RATIONALE: Corticosteroids such as prednisone can lead to elevated blood sugar, especially for patients with diabetes. For this reason, it is useful to monitor the patient's blood sugar. The patient's blood pressure and bowel sounds will not be affected. The hemoglobin A1c will demonstrate average blood sugars over the past three months, which would not evaluate blood sugar since beginning prednisone.

The nurse is caring for a patient with respiratory failure. What measures can the nurse take to prevent the development of stress ulcers in the patient? Select all that apply. 1. Use of parenteral feedings 2. Initiate early enteral nutrition 3. Use smaller tidal volumes 4. Use antiulcer agents such as proton pump inhibitors. 5. Use measures to prevent development of hypotension and shock.

2, 4,5 RATIONALE: Early initiation of enteral nutrition helps to prevent mucosal damage. Prophylactic management includes antiulcer agents such as proton pump inhibitors. Stress ulcer management strategies include correction of predisposing conditions such as hypotension, shock, and acidosis. Use of smaller tidal volumes does decrease the risk of volutrauma but does not affect stress ulcers.

The health care provider requests arterial blood gases (ABGs) for a patient. While performing the test, what should the nurse tell the patient about the test? Select all that apply. 1 The test is done by using pulse oximetry. 2 The test is done to assess changes in acid-base balance. 3 The test is done to assess changes in arterial oxygen (PaO2). 4 The test is done to assess changes in arterial carbon dioxide (PaCO2). 5 The test is done to assess changes in blood urea nitrogen and serum creatinine.

2,3,4 RATIONALE: ABGs are used to assess changes in pH (acid status), PaO2, PaCO2, bicarbonate (base status), and saturation (SaO2). These parameters indicate the oxygen saturation and pH values in arterial blood. Pulse oximetry is used intermittently or continuously to assess arterial oxygen saturation (SpO2) only and does not provide information about other arterial blood gases. ABGs do not provide information regarding blood urea nitrogen and serum creatinine.

A patient sustains an injury to the nose. Surgery is scheduled to repair the deviated nasal septum. The nurse expects to assess what patient symptoms? Select all that apply. 1 Nasal swelling 2 Nasal congestion 3 Nosebleeds 4 Nasal breathing difficulty 5 Redness on the nose

2,3,4 RATIONALE: Common signs that are suggestive of a nasal septal deviation are nasal congestion, frequent nosebleeds, and difficulty in breathing through the nose. As a response to injury, blood circulation increases, causing congestion and frequent nosebleeds. A deviated septum may obstruct the air pathway, leading to difficulty in breathing through the nose. Nasal swelling and redness do not indicate deviated nasal septum but may indicate other injuries to the nose.

A nurse is monitoring a patient with stiff lungs due to acute respiratory distress syndrome (ARDS). What compensatory clinical events may the nurse expect in this patient? Select all that apply. 1 Hypoventilation 2 Respiratory alkalosis 3 Increase in cardiac output 4 Increase in respiratory rate and a decrease in tidal volume 5 Accumulation of carbon dioxide

2,3,4 RATIONALE: The breathing pattern of stiff lung increases carbon dioxide removal, producing respiratory alkalosis. Cardiac output increases in response to hypoxemia, a compensatory effort to increase pulmonary blood flow. Hypoxemia and the stimulation of juxtacapillary receptors in the stiff lung parenchyma initially cause an increase in respiratory rate and a decrease in tidal volume. Hypoventilation occurs after compensatory failure. Carbon dioxide does not accumulate; instead, there may be increased removal of carbon dioxide.

A patient with bacterial pharyngitis is prescribed penicillin. The nurse knows which of the following about this drug? Select all that apply. 1. Is given intravenously 2. Has to be taken for 10 days 3. Is taken several times a day 4. Is taken to prevent rheumatic fever 5. Helps to control accompanying fever 6. Prevents transmitting the disease within 12 hours of taking the first dose

2,3,4 RATIONALE: Oral penicillin needs to be taken several times a day for 10 days to prevent rheumatic fever, which is a sequela to the infection. Patients must take the drug orally for 24-48 hours before they are considered noncontagious. It has no effect on fever.

A patient is recovering from acute respiratory distress syndrome (ARDS). What goals should a nurse have for this patient? Select all that apply. 1. Arterial oxygen (PaO2) above normal limits 2. Oxygen saturation (SaO2) greater than 90% 3. Patent airways 4. PaO2 within normal limits 5. Clear lungs

2,3,4,5 RATIONALE: The goals for a patient recovering from ARDS are SaO2 greater than 90%, patent airways, PaO2 within normal limits for age or baseline values on room air, and clear lungs on auscultation. SaO2 above 90% and PaO2 within normal limits indicate satisfactory arterial oxygenation. Airways should be kept patent and clear off any secretions. Clear lungs indicate that there are no secretions obstructing the airway, and the lungs are functional. PaO2 above normal levels means that the patient is hyperventilating. STUDY TIP: Regular exercise, even if only a 10-minute brisk walk each day, aids in reducing stress. Although you may have been able to enjoy regular sessions at the health club or at an exercise class several times a week, you now may have to cut down on that time without giving up a set schedule for an exercise routine. Using an exercise bicycle that has a book rack on it at home, the YMCA, or a health club can help you accomplish two goals at once. You can exercise while beginning a reading assignment or while studying notes for an exam. Listening to lecture recordings while doing floor exercises is another option. At least a couple of times a week, however, the exercise routine should be done without the mental connection to school; time for the mind to unwind is necessary, too.

A patient has bronchodilator therapy ordered. What information should the nurse include when educating the patient about bronchodilator therapy? Select all that apply. 1 Bronchodilators increase alveolar perfusion. 2 Bronchodilators increase alveolar ventilation. 3 Bronchodilators may worsen hypoxemia at times. 4 Rebound bronchoconstriction may occur with bronchodilators. 5 Side effects of bronchodilators may include tachycardia and hypertension

2,3,5 RATIONALE: Bronchodilators increase alveolar ventilation by relieving bronchospasm. These medications can sometimes worsen hypoxemia by redistributing the inspired gas to areas of decreased perfusion. Side effects of bronchodilators may include tachycardia and hypertension due to activation of adrenergic receptors. Bronchodilators do not increase alveolar perfusion. Bronchodilators do not produce rebound bronchoconstriciton.

A patient is in respiratory distress. Which positions of the patient indicate severity of the distress? Select all that apply. 1 Ability of the patient to breathe while in a lying position indicates moderate distress. 2 Ability of the patient to breathe only in sitting position indicates mild distress. 3 Ability of the patient to breathe only in sitting position indicates severe distress. 4 Ability of the patient to breathe only in a lying position indicates severe distress. 5 Ability of the patient to breathe while lying down but preferring to sit indicates moderate distress.

2,3,5 RATIONALE: The patient's position is an indication of the effort associated with the work of breathing. If the patient is unable to breathe unless he is sitting upright, severe distress is present. Moderate distress is exhibited if the patient is able to lie down but prefers to sit. If the patient is able to lie down, the distress is mild.

A patient is seen at the clinic with a sore throat, and states "I can hardly swallow." The examination of the throat reveals a red and edematous pharynx with patchy exudates. The patient has a temperature of 101º F (38.3º C). The nurse anticipates that the collaborative management will include which of the following? Select all that apply. 1.Antiviral agents to treat influenza. 2.Treatment with antibiotics starting as soon as possible. 3. A throat culture or rapid strep antigen test. 4.Treatment with nystatin (Mycostatin) liquid. 5.Supportive care, including cool, bland liquids.

2,3,5 RATIONALE: These signs and symptoms are indicative of bacterial pharyngitis . A throat culture or rapid strep antigen test will be performed first. Penicillin is the drug of choice for bacterial pharyngitis. Other antibiotics (amoxicillin, azithromycin [Zithromax], cephalosporins) also may be used. Supportive care includes drinking cool, bland liquids and avoiding citrus juices, which may be irritating. The use of antiviral agents or nystatin (an antifungal drug) is not appropriate for bacterial pharyngitis.

An alcoholic patient was involved in a road traffic accident and sustained a minor head injury. What nursing interventions may increase the risk of respiratory failure in such patients? Select all that apply. 1.The administration of steroids to relieve pain and edema 2.The administration of an opioid analgesic to relieve the pain 3.The administration of prophylactic antibiotics to prevent infection 4.The administration of benzodiazepines to prevent development of seizures 5. The administration of nonsteroidal antiinflammatory drugs to relieve pain and edema

2,4, RATIONALE: Opioids and benzodiazepines have a respiratory depressant action in a dose-dependent manner. Since the patient is alcoholic and has sustained a head injury, the chance of going into respiratory failure is higher with the use of opioids and benzodiazepines. Steroids, antibiotics, and nonsteroidal antiinflammatory drugs have no respiratory depressive action. Steroids and nonsteroidal antiinflammatory drugs may improve the patient's condition if respiratory failure is caused by brain stem edema.

A patient with chronic obstructive pulmonary disease (COPD) is receiving oxygen therapy through a mask. Which nursing actions should the nurse perform to ensure proper care of the patient? Select all that apply. 1 Take a chest radiograph. 2 Choose the optimal oxygen device. 3 Assess the need to change intravenous fluids. 4 Assess the need to adjust the oxygen flow rate. 5 Monitor for signs of adverse effects of oxygen therapy.

2,4,5 RATIONALE: Choosing the optimal oxygen device, assessing the need for adjustment in oxygen flow rate, and monitoring signs of adverse effects of oxygen therapy are all duties that the nurse should perform when evaluating the response of the patient to oxygen therapy. Taking a chest radiograph and assessing the need to change intravenous fluids are not relevant to this situation.

A patient has been admitted to the emergency department after complaining of having difficulty breathing for several days. To detect late manifestations of inadequate oxygenation, the nurse would examine the patient for which of these? Select all that apply. 1 Mild hypertension 2 Use of accessory muscles 3 Apprehension and restlessness 4 Cyanosis and cool, clammy skin 5 Pausing for breath between sentences and words

2,4,5 RATIONALE: Late manifestations of inadequate oxygenation include use of accessory muscles, cyanosis and cool, clammy skin, and pausing for breath between sentences and words. Mild hypertension, apprehension, and restlessness are early manifestations of inadequate oxygenation

The nurse is caring for a patient on a ventilator. What measures can a nurse take to prevent development of barotraumas in such a patient? Select all that apply. 1 Initiate enteral nutrition early. 2 Use smaller tidal volumes. 3 Choose peak inspiratory pressure. 4 Choose varying amounts of positive end-expiratory pressure (PEEP). 5 Advise use of measures that increase intrathoracic pressure

2,4,5 RATIONALE: To avoid barotraumas, patients with acute respiratory distress syndrome (ARDS) are often ventilated with smaller tidal volumes and varying amounts of PEEP to minimize oxygen requirements and intrathoracic pressure. Large tidal volume and elevated plateau and peak inspiratory pressures are associated with the risk of barotrauma. Early initiation of enteral nutrition helps in protecting the gut mucosa and has no role in preventing barotraumas.

A patient is admitted to an emergency department with injuries of the face and nose. A nurse notices a clear, pink-tinged discharge from the nostrils of the patient, even after controlling the nasal bleed. What could be the cause of the discharge? 1 Skull fracture 2 Septal deviation 3 Cerebrospinal fluid (CSF) leak 4 Epistaxis

3 RATIONALE: A clear and pink-tinged discharge from the nose even after control of nasal bleeding suggests a cerebrospinal fluid (CSF) leak. It is an emergency situation and can lead to life-threatening complications. Skull fracture is manifested as ecchymosis of the eyes. There is no clear discharge in the event of a septal deviation or epistaxis.

The nurse is caring for a patient with pneumothorax at risk of developing respiratory failure. What is the earliest indication of respiratory failure that the nurse should be observant for? 1. Cyanosis 2. Difficulty in walking 3. Change in mental status 4. Early morning headache

3 RATIONALE: Changes is mental status are often the earliest sign of impending respiratory failure, even before arterial blood gas results are obtained. The brain is very sensitive to even the slightest of variations in oxygen and carbon dioxide levels and acid-base balance. The early manifestations suggestive of inadequate oxygen delivery to the brain may include restlessness, confusion, agitation, and combative behavior. Cyanosis is an unreliable indicator of hypoxemia and is a late sign of respiratory failure because it does not occur until hypoxemia is severe (arterial oxygen [PaO2] of 45 mm Hg or less). Difficulty in walking is unreliable and a late symptom of respiratory failure. A severe morning headache may suggest that hypercapnia occurred during the night and is usually seen in chronic respiratory insufficiency. TEST-TAKING TIP: Answer the question that is asked. Read the situation and the question carefully, looking for key words or phrases. Do not read anything into the question or apply what you did in a similar situation during one of your clinical experiences. Think of each question as being an ideal, yet realistic, situation.

Which observation by the nurse indicates possible hemorrhage in a patient who has just undergone rhinoplasty? 1 Lack of appetite 2 Sudden forgetfulness 3 Excessive swallowing 4 Complaint of dizziness

3 RATIONALE: Frequent swallowing is an indication of hemorrhage after rhinoplasty. Because blood flows down the posterior pharynx into the throat, the patient is forced to swallow frequently. Lack of appetite may be present, but forgetfulness and complaints of dizziness are not complications associated with this procedure.

The nurse cares for a patient with emphysema. What change in the alveolar sacs is the pathophysiological change in the lungs most characteristic of this disease? 1. The alveolar sacs collapse. 2. The alveolar sacs retain CO2. 3. The alveolar sacs are overdistended. 4. The alveolar sacs become filled with fluid

3 RATIONALE: In emphysema the alveolar sacs lose elasticity, become distended with trapped air, and may rupture. This causes obstruction of the alveolar capillary bed and impairs gas exchange. Alveolar sacs do not collapse or become filled with fluid. However, as a result of the overdistention and impaired gas exchange, carbon dioxide will be retained, but this is not the correct answer option because it is not the characteristic of emphysema.

The nurse is monitoring a patient through pulse oximetry. What is monitored through pulse oximetry? 1 Inspired oxygen concentration 2 Expired oxygen concentration 3 Arterial oxygen saturation (SpO2) 4 Venous oxygen saturation

3 RATIONALE: Pulse oximetry is a noninvasive method to determine oxygen saturation levels. It may be used intermittently or continuously to assess SpO2. Pulse oximetry cannot be used to assess inspired oxygen concentration, expired oxygen concentration, or venous oxygen saturation. These can be measured through arterial blood gases measurements or pulmonary artery pressure monitoring.

A 72-year-old patient is in the emergency department with a temperature of 101.4° F and a productive cough with rust-colored sputum. The nurse notifies the health care provider, understanding these findings are indicative of which condition? 1. Tuberculosis 2. Chronic heart failure (CHF) 3. Pneumonia 4. Pulmonary embolus

3 RATIONALE: Rust-colored sputum is indicative of pneumococcal pneumonia. Tuberculosis frequently presents with a dry cough. Pink frothy sputum would be present in CHF and pulmonary edema. A pulmonary embolus would produce hemoptysis.

The nurse determines that a 61-year-old patient with chronic bronchitis has a nursing diagnosis of "impaired gas exchange," after noting an oxygen saturation of 88%. What is an appropriate intervention to add to the care plan? 1 Obtain an arterial blood gas (ABG) 2 Place the patient in the prone position to increase postural drainage 3 Sit the patient upright in a chair leaning slightly forward 4 Administer 6 L oxygen via nasal cannula

3 RATIONALE: The patient with chronic bronchitis can engage in better gas exchange in an upright position leaning slightly forward. Once the patient's oxygen increases, the nurse may obtain an ABG, if requested by the health care provider. Placing the patient in a prone position would further impair gas exchange. Six liters of oxygen is too much oxygen for a patient with chronic bronchitis.

The nurse is caring for a child who is suspected of having cystic fibrosis (CF). The nurse knows that the gold standard of diagnosing this condition is: 1 Genetic test 2 Lung biopsy 3 The sweat chloride test 4 Test for pancreatic insufficiency

3 RATIONALE: The sweat chloride test is the most definitive test in children, but may not be conclusive in adults. Patients with CF secrete four times the normal amounts of sodium and chloride in their sweat. Genetic tests may be used if the sweat test is inconclusive. A lung biopsy is not used to diagnose this condition. Pancreatic insufficiency is related to CF but does not specifically diagnose the disease.

When teaching the patient with chronic obstructive pulmonary disease (COPD) about smoking cessation, what information should be included related to the effects of smoking on the lungs and the increased incidence of pulmonary infections? 1. Smoking causes a hoarse voice 2. Cough will become nonproductive 3. Decreased alveolar macrophage function 4. Sense of smell is decreased with smoking

3 RATIONALE: The damage to the lungs includes alveolar macrophage dysfunction that increases the incidence of infections and thus increases patient discomfort and cost to treat the infections. Other lung damage that contributes to infections includes cilia paralysis or destruction, increased mucus secretion, and bronchospasms that lead to sputum accumulation and increased cough. The patient may be aware already of respiratory mucosa damage with hoarseness and decreased sense of smell and taste, but these do not increase the incidence of pulmonary infection.

What is the priority nursing assessment in the care of a patient who has a tracheostomy? 1. Electrolyte levels and daily weights 2. Assessment of speech and swallowing 3. Respiratory rate and oxygen saturation 4. Pain assessment and assessment of mobility

3 RATIONALE: The priority assessment in the care of a patient with a tracheostomy focuses on airway and breathing. These assessments supersede the nurse's assessments that also may be necessary, such as nutritional status, speech, pain, and swallowing ability.

The nurse attending to an older patient finds the patient coughing. On auscultation, the nurse finds retained pulmonary secretions in the lungs. What nursing interventions should the nurse perform to mobilize secretions? Select all that apply. 1. Advise bed rest for the patient. 2. Limit fluid intake. 3. Encourage the patient to cough. 4. Provide chest physiotherapy. 5. Provide humidified air.

3,4,5 RATIONALE: Retained pulmonary secretions increase the risk of respiratory failure. It is important to mobilize the secretions and clear the airway to facilitate ventilation. Effective coughing is an important measure to move up the secretions and relieve obstruction of the airway. Chest physiotherapy can be used in patients who produce copious amount of sputum and who have a collapsed lung. Humidified air helps to keep secretions liquefied and eases in coughing them out. The patient should not be limited to bed rest; ambulation should be done when possible, since it helps to expand the lungs and clear the secretions. Fluid intake should not be restricted. Adequate fluid intake is required to prevent the secretions from thickening.

Which instructions should the nurse include when teaching self-care to a patient with acute pharyngitis? Select all that apply. 1 Drink citrus juices. 2 Restrict fluid intake. 3 Gargle with warm salt water. 4 Suck on popsicles or hard candies. 5 Use a cool-mist vaporizer or humidifier

3,4,5 RATIONALE: Symptom relief is a major goal of nursing management in a patient with acute pharyngitis. The nurse should instruct the patient to gargle with warm salt water, suck on popsicles or hard candies, and use a cool-mist vaporizer or humidifier. Citrus juices can irritate the throat and should not be recommended. The patient should increase fluid intake to keep the secretions thin so they can be easily expectorated.

A nurse is caring for a patient with respiratory failure. Which medications should be included when planning the therapy for this patient? Select all that apply. 1 Nitroglycerin (Tridil) for bronchospasm 2 Albuterol (Ventolin) for pulmonary congestion 3 Methylprednisolone (Solu-Medrol) for inflammation and bronchospasm 4 Azithromycin (Zithromax) for pain, anxiety, and restlessness 5 Furosemide (Lasix) for pulmonary congestion caused by heart failure

3,5 RATIONALE: Corticosteroids like methylprednisolone (Solu-Medrol) may be used in conjunction with bronchodilating agents for bronchospasm and inflammation. The intravenous diuretic furosemide (Lasix) is used to decrease the pulmonary congestion caused by heart failure. Nitroglycerin (Tridil) will relieve pulmonary congestion caused by heart failure and does not relieve bronchospasm. Albuterol (Ventolin) is used in bronchospasm, not for pulmonary congestion. Azithromycin (Zithromax) is used for pulmonary infection, and not for pain, anxiety, and restlessness.

The patient is prescribed a high-flow oxygen delivery system. Which of the following are high-flow devices? Select all that apply. 1 Nasal cannula 2 Simple face mask 3 Venturi mask 4 Non-rebreather mask 5 Mechanical ventilator

3,5 RATIONALE: Venturi mask and mechanical ventilator are correct because oxygen (O2) delivery systems are classified as low- or high-flow systems. Venturi mask is a high-flow device that delivers fixed concentrations of O2 (e.g., 24%, 28%) independent of the patient's respiratory pattern. Mechanical ventilators are another example of high-flow O2 delivery system. Mechanical ventilation is a means of support until the patient can breathe on his or her own. A nasal cannula, simple face mask, and non-rebreather mask are low-flow devices that deliver O2 in concentrations that vary with the person's respiratory pattern. Because room air is mixed with O2, in low-flow systems, the percentage of O2 delivered to the patient is not as precise as with high-flow systems.

The nurse determines that the patient with chronic obstructive pulmonary disease (COPD) and diabetes mellitus is experiencing adverse effects of albuterol (Proventil) after noting which finding? 1 Blood sugar 139 mg/dL 2 Respiratory rate of 21/minute 3 Temperature of 99.1°F 4 Pulse rate of 102 beats/minutes

4 RATIONALE: Albuterol is a beta-2-agonist that sometimes can cause adverse cardiovascular effects. These would include tachycardia and angina. A pulse rate of 102 indicates that the patient is experiencing tachycardia as an adverse effect. This medication will not affect the blood sugar or the temperature. The respiratory rate is normal and does not indicate any adverse reaction is occurring.

A 71-year-old patient is admitted with acute respiratory distress related to cor pulmonale. Which nursing intervention is most appropriate during admission of this patient? 1 Perform a comprehensive health history with the patient to review prior respiratory problems. 2 Complete a full physical examination to determine the effect of the respiratory distress on other body functions. 3 Delay any physical assessment of the patient and review with the family the patient's history of respiratory problems. 4 Perform a physical assessment of the respiratory system and ask specific questions related to this episode of respiratory distress.

4 RATIONALE: Because the patient is having respiratory difficulty, the nurse should ask specific questions about this episode and perform a physical assessment of this system. Further history taking and physical examination of other body systems can proceed once the patient's acute respiratory distress is being managed.

Which risk factor is an absolute contraindication to receiving lung transplantation? 1 Skin cancer 2 Former smoker 3 Reversible renal failure 4 Chronic active hepatitis B

4 RATIONALE: Chronic active hepatitis, types B and C, are absolute contraindications for lung transplantation. Malignancy, with the exception of skin cancer, is an absolute contraindication. Skin cancer is not an absolute contraindication to receiving lung transplantation. Current smoking is an absolute contraindication. Former smoking is not, however, an absolute contraindication to lung transplantation. Untreatable advanced dysfunction of another major organ system, such as the kidneys, would be an absolute contraindication to lung transplantation. Reversible renal failure is not a contraindication.

A patient with a respiratory condition asks "I know how air gets into my lungs. But how does it get out of them?" The nurse bases the answer on the knowledge that air moves out of the lungs because of 1 Contraction of the accessory abdominal muscles 2 Increased carbon dioxide and decreased oxygen in the blood 3 Stimulation of the respiratory muscles by the chemoreceptors 4 Increase in intrathoracic pressure as elastic recoil of the lungs occurs

4 RATIONALE: In contrast to inspiration, expiration is passive. Elastic recoil is the tendency for the lungs to relax after being stretched or expanded. The elasticity of lung tissue is because of the elastin fibers found in the alveolar walls and surrounding the bronchioles and capillaries. The elastic recoil of the chest wall and lungs allows the chest to passively decrease in volume. Intrathoracic pressure rises, causing air to move out of the lungs. Lungs do not expel air by use of abdominal muscles. An increase in carbon dioxide and decrease in oxygen in the blood do not promote expiration.

The nurse is measuring a patient's pulse oximetry and obtains an SpO2 reading of 88%, and then another reading of 87%. The patient is receiving oxygen per nasal cannula at 2 L/minute and states feeling short of breath. Previous readings were above 94%. What should the nurse do next? 1 Document the findings. 2 Increase the flow of oxygen delivery. 3 Check the pulse oximetry again in an hour. 4 Notify the health care provider for a possible arterial blood gas (ABG) analysis.

4 RATIONALE: The abbreviation SpO2 is used to indicate the oxygen saturation of hemoglobin as measured by pulse oximetry. If there is doubt about the accuracy of the SpO2 reading, obtain an ABG analysis to verify the results. The findings should not be documented as normal. The nurse should not increase oxygen flow without further assessment.

The nurse is caring for a 73-year-old patient who underwent a left total knee arthroplasty. On the third postoperative day, the patient complains of shortness of breath, slight chest pain, and that "something is wrong." Temperature is 98.4° F, blood pressure 130/88, respirations 36/minute, and oxygen saturation 91% on room air. What action should the nurse take first? 1 Notify the health care provider 2 Administer a nitroglycerin tablet sublingually 3 Conduct a thorough assessment of the chest pain 4 Sit the patient up in bed as tolerated and apply oxygen

4 RATIONALE: The patient's clinical picture is most likely pulmonary embolus , and the first action the nurse should take is to assist with the patient's respirations. For this reason, the nurse should sit the patient up as tolerated and apply oxygen before notifying the health care provider. The nitroglycerin tablet would not be helpful and the oxygenation status is a bigger problem than the slight chest pain at this time.

The nurse is completing tracheostomy care. Which of these is the best method for ensuring the fit of tracheostomy ties? 1 Have the respiratory therapist check the ties. 2 Ask the patient if the ties feel comfortable after tying them. 3 Place one finger underneath the ties to ensure they are not too tight around the neck. 4 Place two fingers underneath the ties to ensure they are not too tight around the neck.

4 RATIONALE: When securing tracheostomy ties , place two fingers underneath the ties to ensure that they are not too tight around the patient's neck. The respiratory therapist may not be trained in changing the ties, or may not check them accurately. The patient may not be able to identify if the ties are too tight. One finger beneath the tie is too tight.

When the nurse is explaining treatment to the families, for which patient would noninvasive positive pressure ventilation (NIPPV) be an appropriate intervention to promote oxygenation? 1. A patient whose cardiac output and blood pressure are unstable 2. A patient whose respiratory failure is because of a head injury with loss of consciousness 3. A patient with a diagnosis of cystic fibrosis who currently is producing copious secretions 4. A patient who is experiencing respiratory failure as a result of the progression of myasthenia gravis

4 RATIONALE: NIPPV is most effective in treating patients with respiratory failure resulting from chest wall and neuromuscular disease. It is not recommended in patients who are experiencing hemodynamic instability, decreased level of consciousness, or excessive secretions.

The nurse is caring for a patient with a pulmonary artery (PA) catheter. The nurse knows that this type of catheter is used to obtain: 1. Mixed pulmonic vein blood gas sampling 2. Mixed atrial blood gas sampling 3. Mixed arterial blood gas sampling 4. Mixed venous blood gas sampling

4 RATIONALE: PA catheters are used to sample mixed venous blood gases. It consists of venous blood that has returned to the heart and mixes in the right ventricle. This sampling helps to assess the presence of inadequate tissue oxygen delivery or abnormal oxygen consumption in a patient with impaired cardiac output or hemodynamic instability. It does not assess blood from the pulmonic vein. It is not arterial blood and not blood from the atrial chamber.

The nurse determines that the patient has understood medication instructions about the use of a metered dose inhaler (MDI) when the patient performs which action? 1. Inhales rapidly when activating the inhaler 2. Breathes through the nose with activation of the MDI. 3.Holds the MDI sideways to increase ease of use. 4.Waits one minute between each puff from the MDI.

4 RATIONALE: The patient should wait at least one minute in between puffs to increase medication dispersion throughout the lungs. The patient should inhale slowly, hold the MDI upright, and breathe through the mouth. STUDY TIP: Record the information you find to be most difficult to remember on 3" × 5" cards and carry them with you in your pocket or purse. When you are waiting in traffic or for an appointment, just pull out the cards and review again. This "found" time may add points to your test scores that you have lost in the past.

The nurse provides preprocedure teaching for a patient who is scheduled for bedside thoracentesis. What does the nurse explain is the primary purpose of thoracentesis? 1. Determining the stage of a lung tumor 2. Directly inspecting and examining the pleural space 3. Obtaining a specimen of pleural tissue for evaluation 4. Relieving an abnormal accumulation of fluid in the pleural space

4 RATIONALE: Thoracentesis involves the insertion of a large-bore needle into the pleural space to relieve an abnormal accumulation of fluid in the pleural space. The procedure can significantly relieve symptoms related to this fluid accumulation, such as shortness of breath and discomfort. Thoracentesis cannot reveal the stage of lung cancer or permit direct inspection and examination of the pleural space. It may provide a pleural fluid specimen but not a pleural tissue specimen

A patient is learning to use a metered-dose inhaler (MDI). Which statement by the patient indicates the need for additional teaching? 1 "I will shake the canister before use." 2 "I will hold my breath for as long as I can after inhalation." 3 "I should wait at least one minute between puffs of medication." 4 "I should avoid using a spacer with this inhaler because it is ineffective."

4 RATIONALE: A spacer should be avoided with dry powder inhalers; they are helpful to use with MDIs. Metered dose inhalers require shaking of the canister, the breath should be held to increase absorption of the medication into the lungs, and the patient should wait one to two minutes between puffs.

When administering and reading the tuberculosis (TB) skin test, what measures should the nurse take? 1. Ensure that the injection is given subcutaneously. 2. Do not use a pen around the test area to mark the site 3. Include the reddened flat areas on the skin when measuring the induration. 4. Draw a diagram of the forearm and hand and label the injection sites in the patient's chart.

4 RATIONALE: When a skin test is administered for TB bacilli, the nurse should chart the site of administration by drawing a diagram of the forearm and hand and labeling the injection sites. The nurse should ensure that the injection is given intradermally. The nurse should circle the area with a pen and instruct the patient not to remove the mark. The diameter of the induration should be measured for reading the test. The reddened flat area is not included in the measurement.

A patient is diagnosed with allergy to molds. A nurse is educating the patient on management of rhinitis due to mold allergy. What measures would be helpful to the patient? Select all that apply. 1 Remove pets from the house. 2 Drape the windows well to limit light. 3 Keep plants inside the house in large numbers. 4 Keep closets and basements well lit. 5 Ensure good ventilation to allow ample airflow in the house.

4,5 RATIONALE: Darkness, dampness, and drafts promote the growth of mold spores. Keeping lights on in closets and basement would inhibit the growth of molds. Good ventilation allows for more airflow in the house which reduces humidity and development of molds. Removing pets is not an appropriate suggestion as pets do not carry mold. Draping windows will decrease light in the house and having plants inside the house will increase the humidity, both of which would promote mold growth.

The nurse in the cardiac care unit is caring for a patient who has developed acute respiratory failure. The nurse knows that which medication is being used to decrease this patient's pulmonary congestion and agitation? A. Morphine sulfate B. Albuterol (Ventolin) C. Azithromycin (Zithromax) D. Methylprednisolone (Solu-Medrol)

A RATIONALE: For a patient with acute respiratory failure related to the heart, morphine is used to decrease pulmonary congestion as well as anxiety, agitation, and pain. Albuterol is used to reduce bronchospasm. Azithromycin is used for pulmonary infections. Methylprednisolone is used to reduce airway inflammation and edema. TEST-TAKING TIP: Read every word of each question and option before responding to the item. Glossing over the questions just to get through the examination quickly can cause you to misread or misinterpret the real intent of the question.

The patient has severe chronic obstructive pulmonary disease (COPD). To reduce the size of hyperinflated emphysematous lungs and decrease airway obstruction, the health care provider would most likely recommend what as a surgical option? A.Lung volume reduction surgery (LVRS) B.Lung transplant C.Airway bypass D. Bullectomy

A RATIONALE: Three different surgical procedures have been used in severe COPD. One type of surgery is LVRS. The goal of this surgery is to reduce the size of the lungs by removing the most diseased lung tissue so the remaining healthy lung tissue can perform better. The rationale for LVRS is that reducing the size of the hyperinflated emphysematous lungs results in decreased airway obstruction and increased room for the remaining normal alveoli to expand and function. The procedure reduces lung volume and improves lung and chest wall mechanics. Lung transplantation would not be the first choice for the HCP because it is for carefully selected patients with advanced COPD and this patient's lung volume can be reduced. Bullectomy is for patients with emphysematous COPD who have large bullae. Airway bypass is a procedure still under evaluation.

A new nurse is observed caring for a patient who has a chest tube in place after a partial lobectomy. Which action by the nurse is inappropriate and requires a more experienced nurse to intervene? A. Positioning the patient in a semi-Fowler's position B. Encourage the patient to increase his fluid intake C. Clamping the chest tube while the patient is ambulating to the bathroom D. Administering the patient's prescribed narcotic analgesic before activity

C RATIONALE: Clamping of chest tubes is generally contraindicated. Clamping would allow a buildup of secretions or air in the pleural space and inhibit lung expansion. Patients with chest tubes are usually placed in the semi-Fowler's position for the sake of comfort and ease of breathing. Fluid intake is not a major concern in light of the information provided. Administering the patient's prescribed narcotic analgesic before activity is an expected standard of care for this patient.

A patient comes to the emergency room presenting with dyspnea, tachycardia, violent agitation, tracheal deviation, neck vein distension, and hyperresonance to percussion. Which condition should the nurse suspect? A. Hemothorax B. Flail chest C. Tension pneumothorax D. Cardiac tamponade

C RATIONALE: Tension pneumothorax is the result of increased air in the pleural space and causes shifting of bodily organs and an increase in intrathoracic pressure. The patient usually presents with cyanosis, air hunger, violent agitation, tracheal deviation, neck vein distension, and hyperresonance to percussion. Hemothorax is an accumulation of blood in the pleural space, and the patient usually presents with dyspnea, diminished breath sounds, dullness to percussion, and shock, depending on blood loss. Flail chest is a fracture of two or more ribs, and the patient presents with paradoxical movement of the chest wall and respiratory distress. Cardiac tamponade occurs when blood collects in the pericardial sac, and the patient presents with muffled, distant heart sounds, hypotension, neck vein distension, and increased central venous pressure.

What is an autosomal recessive, multisystem disease characterized by altered function of the exocrine glands? A. Sickle-cell disease B. Tay-Sachs disease C. Spinal muscular atrophy D. Cystic fibrosis (CF)

D RATIONALE: CF is an autosomal recessive, multisystem disease characterized by altered function of the exocrine glands. This defect primarily affects the lungs, pancreas and biliary tract, and sweat glands. Sweat glands excrete increased amounts of sodium and chloride. While sickle-cell disease, Tay-Sachs disease, and spinal muscular atrophy are all autosomal recessive, multisystem diseases, they are not characterized by altered function of the exocrine glands.

When caring for a patient, the nurse assesses tachypnea, a cough, and restlessness. The lung sounds have fine, scattered crackles and the chest x-ray shows new bilateral interstitial and alveolar infiltrates. The nurse is aware that the patient may have an acute lung injury (ALI). In what order does the nurse expect the physiologic changes of ARDS to occur if it happens with this patient? Put a comma and space between each answer choice (1, 2, 3, 4, etc.) 1. Atelectasis 2. Interstitial edema 3. Refractory hypoxemia 4. Surfactant dysfunction 5. Increased inflammatory response 6. Decreased gas exchange surface area

In the injury or exudative phase of acute respiratory distress syndrome (ARDS; one to seven days after acute lung injury) there is interstitial edema and surfactant dysfunction that lead to atelectasis. Widespread atelectasis decreases lung compliance, hyaline membranes form, and refractory hypoxemia occurs. In the reparative or proliferative phase (one to two weeks after acute lung injury) there is an increased inflammatory response, which worsens hypoxia. In the fibrotic phase (two to three weeks after acute lung injury) the lung tissue is remodeled by collagen and fibrous tissue, which decreases the available surface area for gas exchange.


Conjuntos de estudio relacionados

Chapter 13: Meiosis & Sexual Life Cycles

View Set

Unit 4: Transportation and Logistics

View Set

Worksheet 12.3: Uniform Electronic Transactions Act and International Treaties

View Set